contracts

Pataasin ang iyong marka sa homework at exams ngayon gamit ang Quizwiz!

Preexisting Legal Duty

general rule: the modification of an existing contract requires new consideration

Do any third parties have rights and/or obligations under the contract

three types a. third party beneficiary b.assignee of rights c. delegates of duties

•A contracts to build a house for B for $100,000, but A repudiates the contract after doing part of the work and having been paid $40,000. Another builder reasonably charged B $80,000 to finish the house. What are B's damages? - -What would B have received if the contract had been fully performed? House for $100,000 -What did B receive? House for $120,000

-What's the difference? $20,000

Economic duress

1) one party involuntarily accepted the terms of another 2) circumstances permitted no other reasonable alternative, and 3) such circumstances were the result of coercive acts of the other party

Statute of frauds

1. is the contract within the SOF? 2. if so, is the SOF, satisfied? 3. if the contract is subject to the SOF but the SOF is not satisfied, are there non contractual remedies? If the contract is subject to the SOF and the SOF is not satisfied, the contract is unenforceable, assuming the party challenging the contract timely raises the SOF as an affirmative defense.

Timing of contractual components

1. when is an offer effective? upon receipt by the oferee 2. when is a rejection effective? upon receipt by the offeror 3. when is a revocation effective? upon receipt by the offeree 4. when is a counteroffer effective? upon receipt by the offeror 5. when is an acceptance effective? in a manner invited by an offeror; option contract is not until received by the offeror

2. In an action by plaintiff, who was struck in a crosswalk by a car driven by defendant, plaintiff wishes to offer evidence that defendant is a wealthy person. Which of the following statements is most likely correct? A.The evidence might be relevant if plaintiff's action is brought on an intentional tort theory and plaintiff seeks punitive damages. B. Because a wealthy person is not likely to be concerned about the potential financial impact of a tort judgment against her, the evidence is relevant if plaintiff's action is brought on a negligence theory. C. Regardless of the substantive theory supporting plaintiff's action, the evidence is relevant. D. Regardless of the substantive theory supporting plaintiff's action, the evidence is irrelevant.

A

Admissions

A contract which does not satisfy the requirements of subsection 1 but which is valid in other respects is enforceable

Promissory Estoppel

A doctrine that applies when a promisor makes a clear and definite promise on which the promisee justifiably relies; such a promise is binding if justice will be better served by the enforcement of the promise.

Did either party fail to perform its part of the deal, and if so, was that non-performance excused?

A. failure of a condition precedent 1. express condition

Implied warranty of fitness for a particular purpose

was the warranty created? a. all sellers (but usually merchants) b. seller is aware of buyer's particular ise c. buyer relies on seller's experties If the warranty was created, was it disclaimed? a. general disclaimer b. specific disclaimer conspicuous written If the warranty was created and not disclaimed, was it breached?

Implied Warranty of Merchantability

was the warranty created? a. seller is a merchant of this type of goods B. goods are fit for ordinary purposes If the warranty was created, was it disclaimed? a. general disclaimer b. specific disclaimer If the warranty was created and not disclaimed, was it breached?

Confirmations

between merchants if within a reasonable tome a writing in confirmation of the contract and sufficient against the sender is received and the party receiving it has reason to know its contents, it satisfies the requirements of subsection 1 against such party unless written notice of objection to its contents is given within 10 days after it is received.

Buyer in breach, Buyer has goods

breach occurs when: a. buyer accepts goods but doesn't pay b. goods destroyed after risk of loss shifts to buyer damages: a. contract price plus b. incidental damages minus c. expenses saved by breach

Is the deal a contract?

consideration (or consideration substitute No defenses

Consideration

consideration is bargained for legal detriment; it has two parts a. the promisor bargained for an exchange in which b. the promisee suffers a legal detriment by promising to do something (or by doing something) that, but for the contract, he or she is not obligated to do, or b promising not to do something (or in fact not doing something) that, but for the contract he or she is legally entitled to do.

Consideration--sufficiency

consideration must be legally sufficient for there to be a binding contract. To be sufficient, the consideration must be something that has value in the eyes of the law. Insufficient consideration: 1. love and affection 2. performing a service 3. nominal consideration (except for option contracts)

Expectation damages--Certainty

damages are not recoverable for loss beyond an amount that the evidence permits to be established with reasonable certainty New businesses: 1. traditional(minority) lost profits are not recoverable by a new business or enterprise 2. restatement (majority/tn): lost profits are recoverable by a new business or enterprise if proved with reasonable certainty

Liquidated Damages

damages that parties to a contract agree in advance should be paid if the contract is breached; only an amount that is anticipated or actual harm

an offeree's power of acceptance is terminated when the offeree or offeror dies or is deprived of legal capacity to enter into the proposed contract an option contract is not terminated by the death or incapacity of the offeror

death or incapacity of the offeror of offeree

offer

defined as the manifestation of willingness to enter into a bargain, so made as to justify another person in understanding that his or her assent to that bargain is invited and will conclude the bargain

Expectation damages--avoidability

except as stated in subsection 2, damages are not recoverable for loss that the injured party could have avoided without undue risk, burden, or humiliation. 2. the injured party is not precluded from recovery by the rule stated in subsection 1 to the extent that he has made reasonable by unsuccessful efforts to avoid loss

Preexisting legal duty

exceptions restatement (2nd) of contracts modification is fair and equitable in view of circumstances not anticipated by the parties when the contract was made UCC section 2-209 Restatement second of contract section 73 accord and satisfaction

If there was inexcusable non-performance, what are the non breaching party's remedies?

expectation damages or reliance damages

Revocation by the offeror

four situations in which an offer is deemed irrevocable for a period of time: a: option contracts b: offer to enter into a unilateral contract c: merchant's firm offer D: detrimental reliance

Failure of a condition precedent--express conditions

if a condition precedent to a party's performance has not occurred, that party's performance is discharged -express conditions precedent may be satisfied only by strict compliance -the party whose performance is subject to condition precedent must make a good faith attempt to satisfy the condition

Affirmative fraud

if a party's manifestation of assent is induced by either a fraudulent or a material misrepresentation by the other party upon which the recipient is justified in relying, the contract is voidable by recipient.

Undue Influence

if a party's manifestation of assent is induced by influence by the other party, the contract is voidable by the victim. Undue influence is unfair persuasion of a party who is under the domination of the person exercising the persuasion or who by virtue of relation between them is justified in assuming that person will not act in a manner inconsistent with his welfare

dual dispatches

if an offeree rejects an offer by mail and then sends an acceptance, the first document received by the offeror is effective. By contrast, if the offeree accepts by mail but thens sends a rejection, there is a contract, unless the rejection arrives first and the offeror relies on the rejection to his detriment (in which case the offered is estopped from enforcing the contract).

Gap fillers--price

the parties can conclude a contract for sale even though the price is not settled. In such case, the price is a reasonable price at the time for delivery if: a. nothing is said as to price b. the price is left to be agreed by the parties and they fail to agree; or C. the price is to be fixed in terms of some agreed market or other standard as set or recorded by a third person or agency and it is not so set or recorded

Gap fillers--date and mode of deliver

the time for shipment or delivery or any other action under a contract if not agreed upon by the parties shall be a reasonable time. Reasonableness depends on the circumstances, such as: * how complicated the good is to make *past dealings of the parties *the practice of the industry *the needs of the buyer

Warranty of Title

there is in a contract for sale a warranty by the seller that a. the title conveyed shall be goof and its transferred rightful and b. the goods shall be delivered free from any security interest or other lien or encumbrance of which the buyer at the time of contracting has no knowledge

Timely acceptance

to be valid, the offer's acceptance must occur before the offer is terminated. offers may be terminated by 1. revocation by the offeror 2. lapse of time 3. death or incapacity of the offeror of offeree 4. rejection or counteroffer by the offeree

Measuring Expectation Damages

to determine the amount of expectation 1. what would the plaintiff have received if the contract had been fully performed? 2. what did the plaintiff receive? 3. what's the difference?

Physical duress

two types: force or threats of imminent violence or unlawful imprisonment improper threats

If a contract or term thereof is unconscionable at the time the contract is made a court may refuse to enforce the contract, or may enforce the remainder of the contract without the unconscionable term, or may so limit the application of any unconscionable term as to avoid any unconscionable result

unconscionability

Consideration

under American law, not all promises are enforceable in court; only those based on a bargain or exchange are enforceable.

Gap filler-- time and place of payment

under UCC, payment is due at the time and place at which the buyer is to receive the goods

Infancy

unless a statute provides otherwise, a person has the capacity to incur only voidable contractual duties until the person's eighteenth birthday

Express warranty (quality)

was an express warranty created? a. affirmative act by the seller b. basis of buyer's bargain if an express warranty was created, was it disclaimed? If an express warranty was created and not disclaimed, was it breached?

Consideration--motive

the fact that a promise does not of itself induce a performance or return promise does not prevent the performance or return promise from being consideration for the promise

Expectation Measure

the injured party has a right to damages based in his expectation interest as measured by a. the loss in the value to him of the other party's performance caused by its failure or deficiency, plus b. any other loss, including incidental or consequential loss, caused by the breach,less c. any cost or other loss that he has avoided by not having to perform

•7. Olivia Owen and Paul Paige agree that Paul will buy Olivia's house for $300,000. They draft the following memo: I agree to buy Olivia Owen's house at 111 Elm Street, Nashville Tennessee for $300,000. signed/Paul Paige/ • •Does the writing constitute an enforceable contract? •A. No, because it fails to comply with the Statute of Frauds. •B. Yes, but only against Paul. •C. Yes, but only against Olivia. •D. Yes, against both Paul and Olivia.

Answer: B

•. A consumer is test-driving a car of a used car dealership. She hears a sound under the car and asks the salesperson about it. The salesperson says, "Don't worry about that. We stand behind the cars we sell. If anything goes wrong in the next 60 days, we will fix it free of charge." Relieved, the consumer agrees to buy the car. Two days later, the transmission fails. She brings it back to the dealer and tells the manager what the salesperson said. The manager tells her that the salespeople aren't allowed to negotiate terms and all the terms are found in the writing. He points out that the writing effectively disclaims all warranties and conspicuously states that the car is sold "AS IS." He also points out a merger clause that states that there are no promises or understandings other than those found in the writing. Is the consumer likely to recover? •A) Yes, because the evidence does not contradict the writing. •B) Yes, because the parol evidence rule does not apply to consumers. •C) No, because the writing is a partial integration and the evidence does not contradict the writing. •D) No, because the writing is a full integration.

D

Is SOF satisfied

common law: any writing

•Installment Contracts

(1) An "installment contract" is one which requires or authorizes the delivery of goods in separate lots to be separately accepted.... •(2) The buyer may reject any installment which is non-conforming if the non-conformity substantially impairs the value of that installment and cannot be cured.... •(3) Whenever non-conformity or default with respect to one or more installments substantially impairs the value of the whole contract there is a breach of the whole.... • •Unless the contract expressly calls for installments, the UCC presumes a single delivery contract. •

Injunctive relief

(1) a promise to render personal service will not be specifically enforced (2) a promise to render personal service exclusively for one employer will. not be enforced by an injunction against serving another if its probable result -will be to complete a performance involving personal relations the enforced continuance of which is undesirable or -will be to leave the employee without other reasonable mean of making a living

Expectation damages--foreseeability

(1) damages are not recoverable for loss that the party in breach did not have reason to foresee as a probable result of the breach when the contract was made (2) loss may be foreseeable as a probable result of a breach because it follows from the breach -in the ordinary course of events, or -as a result of special circumstances, beyond the ordinary course of events, that the party in breach had reason to know (3) a court may limit damages for foreseeable loss by excluding recovery for loss of profits, by allowing recovery only for loss incurred in reliance, or otherwise if it concludes that in the circumstances justice so requires in order to avoid disproportionate compensation.

Laura resolved to pull a prank on billy. She revealed to her friends that she would pretend to offer Billy $300 for his old laptop. She knew Billy needed the money and that his computer has a market value of just $50. When Laura offered Billy $300 for his computer, she explained her interest by saying, "I think there's a collector's item underneath all those dents and scuffs." Billy blushed but immediately replied, "I accept your offer and will deliver the computer to you tomorrow." Laura said, "thanks," and waled away. When Billy presented Laura with the computer the next day, Laura and her friends busted into laughter. "you doofus!" Laura said, "noboddy wants your junky laptop! I was pulling a prank and you fell for it." Billy stormed off. Based on the facts, it is most likely that

(a) a contract exist because Billy commenced performance (b) a contract exists because Billy reasonably believed that Laura made a serious offer, which he accepted (c) no contract exists because Laura did not intend to make an offer (d) no contract exists because Laura told her friends that the offer was made in jest. Answer: B

•. An unsophisticated elderly patient in a hospital needed insulin immediately to treat her diabetes. The hospital forced her to sign a contract relinquishing any right to sue for physical injuries that might develop from the medicine and the price was three times as high as the price available in public drug stores. The insulin was defective causing her death. If her personal representative sues the hospital and it defends on the basis of the insulin contract relinquishment, the result is •(A) The court (judge) would likely hold the contract relinquishment unconscionable. •(B) The jury would likely hold the contract relinquishment unconscionable. •(C) Dismissal because the estate can only prove procedural unconscionability not substantive unconscionability. (D) Dismissal because the estate can only prove substantive unconscionability not procedural unconscionability

A

•. Elda, the aged mother of Alice and Barry, both adults, wished to employ a live-in companion so that she might continue to live in her own home. Elda, however, had only enough income to pay one half of the companion's $2,000 monthly salary. Learning of their mother's plight, Alice and Barry agreed with each other in a signed writing that on the last day of January and each succeeding month during their mother's lifetime, each would give Elda $500. Elda then hired the companion. Alice and Barry made the agreed payments in January, February, and March. In April, however, Barry refused to make any payment and notified Alice and Elda that he would make no further payments. For this question only, assume that there is a valid contract between Alice and Barry and that Elda has declined to sue Barry. Will Alice succeed in an action against Barry in which she asks the court to order Barry to continue to make his payments to Elda under the terms of the Alice-Barry contract? •(A) Yes, because Alice's remedy at law is inadequate. •(B) Yes, because Alice's burden of supporting her mother will be increased if Barry does not contribute his share. •(C) No, because a court will not grant specific performance of a promise to pay money. •(D) No, because Barry's breach of contract has caused no economic harm to Alice.

A

•1. Seisin and Vendee, standing on Greenacre, orally agreed to its sale and purchase for $5,000, and orally marked its bounds as "that line of trees down there, the ditch that intersects them, the fence on the other side, and that street on the fourth side." In which of the following is the remedy of reformation most appropriate? •A. As later reduced to writing, the agreement by clerical mistake included two acres that are actually beyond the fence. •B. Vendee reasonably thought that two acres beyond the fence were included in the oral agreement but Seisin did not. As later reduced to writing, the agreement included the two acres. •C. Vendee reasonably thought that the price orally agreed upon was $4,500, but Seisin did not. As later reduced to writing, the agreement said $5,000. •D. Vendee reasonably thought that a dilapidated shed backed up against the fence was to be torn down and removed as part of the agreement, but Seisin did not. As later reduced to writing, the agreement said nothing about the shed.

A

•A gambler owed his uncle $9,000, which was due on January 1. On January 15, the gambler offered to pay the uncle $8,000 if he would agree to accept the amount in full satisfaction of the $9,000 debt. The uncle agreed and the gambler paid him the $8,000. If the uncle then sues the gambler for $1,000, the uncle will: •A. Win, because the gambler had an obligation to pay $9,000 on January 1. •B. Lose, because of the uncle's agreement to accept $8,000. •C. Lose, because there was an accord and satisfaction. •D. Lose, because the uncle agreed to the $8,000 after the January 1 due date.

A

•A large wholesaler of European clothing sent out a circular to all of its customers advertising fine quality coats for sale at a price of $300 each when purchased in lots of 100 coats. The circular also stated that payment for each lot was to be made in three $10,000 installments, the first payment due within 10 days of delivery and the subsequent installments within 40 and 70 days of delivery, respectively. The day after receiving the circular, a retailer sent the wholesaler a standard order form with price, quantity, and delivery date terms filled in by the retailer, clearly indicating that the retailer was ordering one lot of 100 coats. The wholesaler shipped the coats to the retailer, and they arrived on September 10. The retailer accepted delivery of the coats and, on September 11, mailed to the wholesaler a check for $29,400. The retailer enclosed a letter with the check stating, in relevant part, "By cashing the enclosed check in the amount of $29,400, we assume that you accept our standard policy of taking a 2% discount for early payment." The wholesaler received the check on September 13, read the letter, and cashed the check the next day. What is the legal effect of the wholesaler's cashing of the retailer's check? •A. The entire debt is discharged, because there has been an accord and satisfaction. •B. The entire debt is discharged because both parties are merchants. •C. The retailer remains liable for the unpaid portion of the debt, because the discount provision was never accepted as part of the contract. •D. The retailer remains liable for the unpaid portion of the debt because the check did not state "payment in full" or words to that effect.

A

•A seller agrees to deliver 1,000 widgets by June 1. The widgets are not tendered until June 8. The buyer accepts delivery of the widgets and then informs the seller that it is seeking damages for the late delivery. Assuming the buyer suffered some loss, is the buyer entitled to damages? •A. Yes, because the seller did not tender the widgets in conformity with the contract. •B. No, because the breach was immaterial. •C. No, because time is not of the essence. •D. No, because the buyer waived its rights by accepting the late delivery.

A

•A seller's form conspicuously states on the front page, "THERE ARE NO WARRANTIES, EXPRESS OR IMPLIED." Is that language sufficient to disclaim the implied warranty of merchantability? •A. No, because it does not use the word "merchantability." •B. No, because it is not conspicuous. •C. No, because it is disclaimed only when the implied warranty of fitness for a particular purpose is disclaimed. •D. Yes, because the disclaimer is in writing and conspicuous.

A

•Brenda and Sarah are law students. Brenda tells Sarah (a senior editor of the law review) that she needs word processing software to write her law review article. Sarah says, "I just bought new software, so I'll sell you my old software." Brenda buys the software and discovers that although it otherwise works perfectly well, it does not create footnotes and is therefore not useful for writing her law review article. Assuming Article 2 applies to this transaction, has Sarah breach the implied warranty of fitness for a particular purpose? •A. Yes, because she impliedly promised that the software would create footnotes. •B. Yes, because the software is not fit for the ordinary purpose of word processing. •C. No, because neither party is a merchant. •D. No, because no warranty was given in writing.

A

•Mermaid owns an exceptionally seaworthy boat that she charters for sport fishing at a $500 daily rate. The fee includes the use of the boat with Mermaid as the captain, and one other crew member, as well as fishing tackle and bait. On May 1, Phinney agreed with Mermaid that Phinney would have full-day use of the boat on May 15 for himself and his family for $500. Phinney paid an advance deposit of $200 and signed an agreement that the deposit could be retained by Mermaid as liquidated damages in the event Phinney canceled or failed to appear. On May 15 at 1:00 a.m., the Coast Guard issued offshore "heavy weather" warnings and prohibited all small vessels the size of Mermaid's boat from leaving the harbor. This prohibition remained in effect all day. Phinney did not appear at all on May 15, because he had heard the weather warnings on his radio. Which of the following is an accurate statement? •A. The contract is discharged because of impossibility, and Phinney is entitled to return of his deposit. •B. The contract is discharged because of mutual mistake concerning an essential fact, and Phinney is entitled to return of his deposit. •C. The contract is not discharged, because its performance was possible in view of the exceptional seaworthiness of Mermaid's boat, and Phinney is not entitled to return of his deposit. •D. The contract is not discharged, and Phinney is not entitled to return of his deposit, because the liquidated-damage clause in effect allocated the risk of bad weather to Phinney.

A

•Same facts as the prior slide. Assume that in response to Legs's September 15 announcement, Team, on December 31, hires another player to replace Legs. On the next day, January 1, Legs reports for play as required by his contract. Team Owner advises Legs that he is not welcome to play, that he will not be paid a salary and that he is "fired." Which party has an action against which party? •A. Team has an action against Legs for total breach. •B. Legs has an action against Team for total breach. •C. Each party has an action against the other for total breach. •D. Neither party has an action against the other for breach.

A

•Tom wants to change the oil in his automobile, and to do so, he needs to purchase an oil filter. He goes to the auto supply store and tells the clerk that he needs to change the oil in his vehicle and wants an appropriate oil filter for his make and model of vehicle. The clerk recommends oil filter XYZ-2. Tom purchases the oil filter, takes it home, and puts it on his vehicle. The oil filter (which has no defects) isn't suited for the vehicle and, as a result, the vehicle is damaged. Tom can sue the auto supply store successfully based on breach of •A. implied warranty of fitness for a particular purpose. •B. implied warranty of merchantability. •C. implied warranty of merchantability and implied warranty of title. •D. implied warranty of title.

A

•Ven owned Goldacre, a tract of land, in fee simple. Ven and Pur entered into a written agreement under which Pur agreed to buy Goldacre for $100,000, its fair market value. The agreement contained all the essential terms of a real estate contract to sell and buy, including a date for closing. The required $50,000 down payment was made. The contract provided that in the event of Pur's breach, Ven could retain the $50,000 deposit as liquidated damages. Before the date set for the closing on the contract, Pur died. On the day that Addy was duly qualified as administrator of the estate of Pur, which was after the closing date, Addy made demand for return of the $50,000 deposit. Ven responded by stating that he took such demand to be a declaration that Addy did not intend to complete the contract and that Ven considered the contract at an end. Ven further asserted that Ven was entitled to retain, as liquidated damages, the $50,000. The reasonable market value of Goldacre had increased to $110,000 at that time. Addy brought an appropriate action against Ven to recover the $50,000. In answer, Ven made no affirmative claim but asserted that he was entitled to retain the $50,000 as liquidated damages as provided in the contract. In such lawsuit, judgment should be for •A. Addy, because the provision relied upon by Ven is unenforceable. •B. Addy, because the death of Pur terminated the contract as a matter of law. •C. Ven, because the court should enforce the express agreement of the contracting parties. •D. Ven, because the doctrine of equitable conversion prevents termination of the contract upon the death of a party.

A

Incompetency

A person has no capacity to incur contractual duties if his or her property is under guardianship by reason of an adjudication of mental illness or defect A person incurs only voidable contractual duties by entering into a transaction if by reasonable of mental illness or defect a. he or she is unable to understand in a reasonable manner the nature and consequences of the transaction or b. he or she is unable to act in a reasonable manner in relation to the transaction and the other party has reason to know of his condition

Intoxication

A person incurs only voidable contractual duties by entering into a transaction if the other party has reason to know that by reason of intoxication a. he or she is unable to understand in a reasonable manner the nature and consequences of the transaction, or b. he or she is unable to act in a reasonable manner in relation to the transaction

Parol Evidence Rule

A substantive rule of contracts under which a court will not receive into evidence the parties' prior negotiations, prior agreements, or contemporaneous oral agreements if that evidence contradicts or varies the terms of the parties' written contract.

Suretyship contracts

A surety's promise to a creditor to answer for the debt of the principal debtor is within the SOF, unless the primary (or main) purpose of the surety's promise was to benefit the surety.

•. A corporation whose subsidiaries include a major hotel chain planned to build a new hotel and advertised for bids to build the hotel within the next six months. Four bids were received, for $17 million, $17.2 million, $17.4 million, and $15 million. The corporation's chief financial officer reviewed the bids, then emphatically told the corporation's chief executive officer ("CEO") that there was "no way" the low bidder could make a profit on the $15 million bid. The CEO made no response. In fact, the builder had stayed up for 72 hours without sleep preparing the bid for the hotel project and had neglected to include the plumbing expenses in the bid. Typically, the cost of plumbing, including the shop's profit, would have been about $2 million. Shortly after the $15 million contract was signed by the CEO and the builder, the builder discovered his mistake and telephoned the CEO to tell her that he had forgotten to include the cost of plumbing, adding that he would normally charge $2 million for plumbing. The CEO agreed to pay the additional $2 million, but this arrangement was never reduced to writing. After the builder completed the project on time, the CEO sent him a check for only $15 million. Can the builder compel the CEO to tender the additional $2 million? •A. Yes, because the CEO was on notice of the builder's mistake. •B. Yes, because the builder relied to his detriment on the CEO's promise. •C. No, because the builder had a preexisting legal duty to complete the project for $15 million. •D. No, because evidence of the agreement to pay the additional $2 million is barred by the Statute of Frauds.

A.

•A builder contracted to build a house for a newly married couple. Terms of the contract provided that the builder would receive the contract price when the building was fully completed. Just when the builder had completed one-half of the structure, a tornado struck the area and demolished the building. What is the builder entitled to recover from the couple under the contract? •A. Nothing. •B. One-half of the contract price. •C. One-half of the fair market value of what remains of the house. •D. Cost of materials and reasonable labor costs.

A.

Bobby, age 17, enters into an oral agreement to purchase Sally's 1974 ford pinto for $500. Prior to either party performing, sally repudiates. Bobby brings suit for breach of contract. If sally wins. it will be becuase of:

A. Bobby's age B. Sally's age C. Lack of cocnsideration D. SOF Answer: D

Buyer faxed the following signed message to seller, his longtime widget supplier: "Urgently need blue widgets. Ship immediately three gross at your current list price of $600." Upon receipt of the fax, seller shipped three gross of red widgets to buyer and faxed to buyer the following message: "Temporarily out of blue. In case red will help, I am shipping three gross at the same price. Hope you can use them." Upon buyer's timely receipt of both the shipment and seller's fax, which of the following best describes the rights and duties of buyer and seller?

A. Buyer may accept the shipment, in which case he must pay seller the list price, or he must reject the shipment and recover from seller for total breach of contract. B. Buyer may accept the shipment, in which case he must pay seller the list price, or he may reject the shipment, in which case he has no further rights against seller. C. Buyer may accept the shipment, in which case he must pay seller the list price, less any damages sustained because of the nonconforming shipment, or he may reject the shipment and recover from seller for total breach of contract, subject to seller's right to cure. D. Buyer may accept the shipment, in which case he must pay seller the list price, less any damages sustained because of the nonconforming shipment, or he may reject the shipment provided that he promptly covers by obtaining conforming widgets from another supplier. Answer: B

Loomis, the owner and operator of a small business, encourages "wellness" on the part of his employees and supports various physical fitness programs to that end. Learning that one of his employees, Graceful, was a dedicated jogger, Loomis promised to pay her a special award of $100 if she could and would run one mile in less than six minutes on the following Saturday. Graceful thanked him, and did in fact run a mile in less than six minutes on the day specified. Shortly thereafter, however, Loomis discovered that for more than a year Graceful had been running at least one mile in less than six minutes every day as a part of her personal fitness program. He refused to pay the $100. In an action by Graceful against Loomis for breach of contract, which of the following best summarizes the probable decision of the court?

A. Loomis wins, because there is a compelling inference that Loomis' promise did not induce Graceful to run the specified mile. B. Loomis wins, because Graceful's running of the specified mile was beneficial, not detrimental, to her in any event. C. Graceful wins, because running a mile in less than six minutes is a significantly demanding enterprise. D. Graceful wins, because she ran the specified mile as requested, and her motives for doing so are irrelevant. Answer: D

A wholesale seller of DVD players emailed a message to the owner of a retail electronic store, stating that he had recently received a new shipment of 200 X brand DVD players that were available for sale to the store owner. In a return email, the store owner accepted the offer to purchase the Dvd players, but added that it would be easier, given his limited space, if the wholesaler delivered 50 of the DVD players each month for the next four months. The wholesalers emailed back to the store owner, saying that he would only ship the entire order of 200 DVD players now. If the wholesaler then tenders the 200 conforming DVD players, the store owner:

A. May reject the entire deliver B. Must accept the entire delivery C. May demand the wholesaler deliver 50 DVD players a month for the next four months D. Must accept or reject the entire delivery Answer: B

A company contracted with a builder to construct a new corporate headquarters for a fixed price of $100m. At the time of the contract, structural steel was widely available and was included in the contract as a $6 m. item. before work began on the project, tornado damage shut down the production facility of the biggest structural steel supplier in the country, and the price of structural steel increased by 20% as a result. The builder informed the company of the steel price increase, and the parties then orally agreed to increase the project price to $101 m. The builder proceeded with construction and delivered the project on time. The company paid the builder $100m but refused to pay the additional $1 mil. If the builder sues the company for $1 mil., is the builder likely to prevail?

A. No, because the modification was never reduced to a writing signed by the party to be charged B. No, becuase there was no consideration for the modification of the contract C. Yes, because the company's promise was supported by consideration. D. Yes, becuase the modification was fair and equitable in view of the unanticipated increase in the price of structural steel Answer: D

Buyer mailed a signed order to Seller that read: "Please ship us 10,000 widgets at your current price." Seller received the order on January 7 and that same day mailed to Buyer a properly stamped, addressed, and signed letter stating that the order was accepted at Seller's current price of $10 per widget. On January 8, before receipt of Seller's letter, Buyer telephoned Seller and said "I hereby revoke my order." Seller protested to no avail. Buyer received Seller's letter on January 9. Because of Buyer's January 8 telephone message, Seller never shipped the goods. Under the relevant and prevailing rules, is there a contract between Buyer and Seller as of January 10?

A. No, because the order was an offer that could be accepted only by shipping the goods; and the offer was effectively revoked before shipment. B. No, because Buyer never effectively agreed to the $10 price term. C. Yes, because the order was, for a reasonable time, an irrevocable offer. D. Yes, because the order was an offer that Seller effectively accepted before Buyer attempted to revoke it. Answer: D

A buyer of goods sends a purchase order to a seller on its form. The boilerplate on the form states that the seller is liable for consequential damages. The seller responds with an acknowledgment form that contains boilerplate that states that the seller is not liable for consequential damages. In addition, the seller's form states, acceptance is expressly made conditional on assent to the additional or different terms in this acknowledgment. After the forms are exchanged, the seller ships the goods and the buyer pays for them. The buyer then suffers consequential damages because of a breach by the seller. Is the seller liable for consequential damages?

A. No, because there is no contract between the parties B. No, because the seller's form governs C. Yes, because the buyer's form governs D. Yes, because the UCC provides for consequential damages Answer: D

A buyer of goods (retailer)sends a purchase order to seller (Manu) on its form. The seller responds with an acknowledgement form that has additional and different terms. In addition, the seller's form states, acceptance is expressly made conditional on assent to the additional or different terms in this acknowledgment." When t receives the form, the buyer refuses to go through with the deal. Is the buyer in breach?

A. No, because under the common law there was a counteroffer that was not accepted B. No, because the UCC says there was no acceptance and therefore no contract C. Yes, because the UCC says there is an acceptance and therefore a contract D. Yes, because there was acceptance by conduct Answer: B

Homeowner hired incompetent, a professional carpenter, to construct a new deck on homeowner's house. Under the arrangement, Incompetent was to provide all the materials (the wood, nails, etc). and to complete construction for a price of $2,000. The materials themselves cost only $300. After incompetent finished and received payment, Homeowner had a party. When Doug Drinker, one of the guest, walk out onto the deck, the deck collapsed and drinker was severely injured. If drinker sues incompetent for breach of the UCC's implied warranty of merchantability and implied warranty of fitness for a particular purpose, Drinker most likely will

A. Prevail on both warranties B. lose on the implied warranty of merchantability because of fitness for a particular purpose c. prevail on the implied warranty of merchantability because incompetent is a merchant, but lose on the implied warrant of fitness for a particular purpose d. lose on both warranties Answer: d

On October 1, a siding installer sent a letter to a builder stating, I am offering to install siding on one or up to all of the 12 houses that you are currently building. This offer remains open until November 1." On October 13, the builder emailed the installer, "we accept your offer respect to the house on Main Street." There was only one house on Main Street that the builder was constructing, so the installer promptly began the siding installation. On October 25, the other work and so would not able to install siding on the builder's other houses, The builder thereafter faxed the installer, "we would like you to do the siding on the other houses." In action by the builder against the installer for breach of contract, the builder will probably:

A. Succeed, because the installer had promised that the offer would remain open until November 1. B. Succeed, because the installer's attempted revocation was by telephone. C. Not succeed, becuase the builder's power of acceptance was terminated by the installer's contract with another party. D. Not succeed, because the builder's power of acceptance was terminated by effective revocation. Answer: D

In a contract for the sale of good, a merchant offeror's form states, this contract is governed by the law of Texas." the merchant offeree's form states, this contract is governed by the law of Vermont." In a jurisdiction that employee the knockout rule, which jurisdiction supplies the governing law?

A. Texas, because the offeror should be able to choose the applicable law B. Texas, because Vernmont is materially different C. Vernmont, becuase it is not materially different form texas D. Whichever state prevails when the UCC choice of law rule are applied. Answer: D

A contractor sends a company its bid on a construction job. The bid consists of specifications and prices, followed by a number of boilerplate provisions. The company responds by writing up the price and specifications on its own forms, which it sends to the contractor. The contractor completes the project and the company inspects it after 25 days. The company finds some unfinished work and demands payment for it. The contractor claims that he is not responsible for the unfinished work because the form he sent says that inspection must be completed within 20 days after construction, and the company did not timely inspect. However, the company's form indicates that it has 30 days after completion to inspect. Who is responsible for the unfinished work?

A. The owner because the contractor's form governs B. the contractor, because the company's form governs C. neither, because there is no agreement on this term D. both terms are knocked out and the party who would be responsible is supplied by trade usage or by a court Answer: B

On June 1, T, a wholesaler of widgets, received a purchase- order form from W, a retailer of widgets, in which W ordered 10 anti-recoil widgets for delivery no later than August 30 at a total price of $1,000, as quoted in T's catalog. On June 2, T mailed to W its own form, across the top of which it was written, we are pleased to accept your order. This acceptance is expressly made conditional on your assent to the additional or different terms contained herein. T's form contained the same same terms as W's from except for an additional printed clause that provided for a maximum liability of $10 for any breach of contract by T. As of June 5, when W received T's acceptance form, which of the following is an accurate statement concerning the legal relationship between T and W?

A. There is no contract because the liability-limitation clause in T's form is a material alteration of W's offer. B. There is no contract unless W thereafter consents to the liability-limitation clause in T's form C. There is an enforceable contract whose terms do not include the liability-limiting clause in T's form becuase such clause is a material alteration of Ws offer D. There is an enforceable contract whose terms do not include the liability limiting clause in T's form because in T's form because W did not consent to the liability limitation clause in T's form Answer: B

On December 15, lawyer received from stationer, inc. a retailer of office supplies, an offer consisting of its catalog and a signed letter stating, we will supply you with as many of the items in the enclosed catalog as you order during the next calendar year. We assure you that this offer and the prices n the catalog will remain firm throughout the coming year. No other correspondence passed between stationer and lawyer until the following April 15 (4 months later), when stationer received from lawyer a faxed order for the 100 reams of your paper, catalog item #101. Did lawyer;s April 15 fax constitute an effective acceptance of stationer's offer at the prices specified in the catalog?

A. Yes because stationer had not revoked its offer before April 15 B. yes, becuase. one-year option contract had been created by stationer's offer C. no, becuase under applicable law the irrevovability of stationer's offer was limited to a period of three months D no, recuasse lawyer did not accept stationer's offer within a reasonable time. Answer: A

Walmart purchases a full page ad in the weekly newspaper. In this week's ad, Walmart lists a Samsung DVD player for $75. The newspaper, however, made a mistake. The advertisement should have listed the DVD player at $475. Upon seeing the ad, buyer foes to the nearest Walmart, walks up to the electronics department, shows the ad to the clerk, and before the clerk can say a word, buyers says, "I accept." At this point, do the parties have a contract?

A. Yes, because the buyer accepted before Walmart revoked the offer B. no, if Walmart attempted to retract the ad within a reasonable time after its publication C. no, because such contracts must be in writing D. no, recuasse buyer's attempted acceptance was nothing but an offer, which Walmart has yet to accept. Answer: D

Best music, inc. offers to buy from sound products corporation (SPC) 100,000 blank Sony cds. without notifying best, spc timely ships 100,000 RCA CDS. This shipment is:

A. an acceptance of Best's offer B. a breach of the parties contract C. both (a) and (b) D. counteroffer Answer: C

A dairy farmer hired a local company to assemble milking machines that the farmer had purchased. The written contract between the parties provides that the company would assemble and install the milking machines in the farmer's dairy barn within 30 days, in time for the arrival of additional cows, and the farmer agreed to Pay the company $10,000. Three weeks into the job, the company realized that it would lose $2,500 on the job, due to a new wage arrangement forced on the company by its employees union after the contract was executed. The company approached the farmer and told him that the job could not be completed for less than $12,500. After some discussion, the farmer and the company executed an agreeement obligating the farmer to pay an additional $2,500 upon completion of the job. The company completed the work on time, but the farmer now refuses to pay the additional $2,500. In a suit by the company against the farmer, which of the following would be the farmer's strongest position?

A. he has no duty to pay the company more than $10,000, because this was a contract for services and the modification was not supported by consideration. B. The modification is voidable because the company knew that the farmer needed the machines up and running in 30 days and took advantage of his duress. c. The company's mistake regarding the cost of providing its services is not grounds for voiding the original contract D. during initial contract negotiation, the company assured the farmer that the milking machines would be assembled and installed for no more than $10,000. Answer: A

Same facts, as On December 15, lawyer received from stationer, inc. a retailer of office supplies, but for this question only assume that on January 15, having at that time received no reply from lawyer, stationer notified lawyer that effect feb. 1, it was increasing the prices of certain specified items in its catalog. is the price increase effective with respect to catalog orders stationer receives from lawyer during the month of February?

A. no, because stationer's original offer, including the price term, became irrevocable under the doctrine of promissory estoppel b. no, recuasse stationer is a merchant with respect to office supplies; and its original offer, including the price term, was irrevocable throughout the month of february c. yes, because stationer received no considereation to support its assurance tat it would not increase prices D. yes, because the period for which stationer gave assurance that it would not raise prices was longer than three months Answer: B

On September 15, a highlighter manufactuer faxed a large office supply company offering to sell the supply company 50,000 highlighters for 25,000k. The supply company faxed back the following communication: we accept your offer. Please box 125 highlighters per case in post consumer cardboard shipping boxes. Assuming the existence of a valid contract, its terms would include:

A. only those items set forth in the manufacturer's fax of September 15, because the manufacturer did not assent to any enlargement of the shipping terms. B. All terms set forth in the manufacturer's offer plus consistent additional terms proposed in the office supply company's acceptance C. All terms set forth in the manufacturer's offer plus those in the office supply company's attempted acceptance that did not amount to a material alteration of the manufacturer's offer. D. all the terms set forth in the manufacturer's offer plus all those in the odic supply company's purported acceptance that did not amount to a material alteration of the manufacturer's offer and to which the manufacturer did not object within a reasonable time. Answer: D

Fruitko, inc. ordered from Orchard, inc., 500 bushels of No. 1 royal fuzz peaches, at a specified price, for prompt shipment." Orchard promptly shipped 500 bushels, but by mistake shipped No.2 royal fuzz peaches instead of No. 1. the error in shipment was caused by the negligence of orchard's shipping clerk. Which of the following best states Fruitko's rights and duties upon delivery of the peaches?

A. orchard's shipment of the peaches was a counteroffer and Fruitko can refuse to accept them B. Orchard's shipment of the peaches was a counteroffer but, since peaches are perishable, Fruitko, if it does not want to accept them, must reship the peaches to orchard in order to mitigate Orchard's losses. Fruitko must accept the peaches because a contact was formed when Orchard shipped them D. Although a contract was formed when Orchard shipped the peaches, Fruitko does not have to accept them. Answer: D

On July 26, a manu of computer accessories received a purchase order form from a retailer who ordered 2,000 ergonomic mouse pads for delivery no later than Sept. 1 for a total price of $10,000 as quoted in the Manu's catalog. Two days later, the Manu faxed its own purchase order acceptance form to the retailer, who was a first time customer. This form stated that it was an acceptance of the specified order, was signed by the manu's shipping manager, and contained all of the terms of the retailer's form, but it also contained an additional printed clause stating that all disagreements under this sale are subject to arbitration by the American Arbitration Association. Assuming no further communication between the parties, which of the following is an accurate statement of the legal relationships between the Manufacture and the retailer?

A. there is an enforceable contract between the parties whose terms probably do not include the arbitration clause in the manufacture's form. B. There is an enforceable contract between the parties whose terms include the arbitration clause in the manufacturer's form C. There is no enforceable contract between the parties because the manufacturer's form constituted a rejection of the retailer's offer and a counteroffer by the manufacturer. D. There is no enforceable contract between the parties because the manufacturer's form added an additional term that materially altered the terms of the retailer's offer. Answer: A

On June 1, Topline Wholesale, inc. received a purchase order form from wonder good, inc. a retailer and new costumer, in which the latter ordered 1,000 anti-recoil widgets for delivery no later than August 30 at a delivered total price of $10,000, as quoted in TOpline's current catalog. On June 2, Topeline mailed to wonder Good its own form, across the top of which Topeline's president had written, we are pleased to accept your order." the form contained th same terms as wonder goods form except for an additional printed clause in Toplines form that provided for a maximum liability of $100 for any breach of contract by Topline. As of June 5, when wonder-good received Topleine's form, which of the following is an accurate statement concerning the legal relationship between Topeline and Wonder good?

A. there is no contract, because the liability-limitation clause in Topeline form is a material alteration of wonder-good offer B. There is no contract, because Wonder Good did not consent to the liability-limitation in Topline's form C. there is an enforceable contract whose terms include thelaibility limitation clause in Topeline's form because liquidation of damages is expressly authorized by the UCC D. there is an enforceable contract whose terms doe not include the liability limitation clause in towlines form Correct answer: D

Accord and Satisfaction

An agreement by the parties to a contract to accept a different type of performance by one party (the accord) and that party so performs (the satisfaction) Part payment of debt: if a creditor agrees to take less than full payment in exchange for a release, may the creditor seek the full payment? 1. if the debt is due and undisputed in amount, part payment does NOT constitute consideration for the release and thus the debtor owes the remainder 2. but if the deb is not yet due (and the debtor agrees to pay early) or if there is a good faith dispute as to the amount of the dept, part payment would constitute consideration for a release

Termination by counteroffer

An offeree's power of acceptance is terminated when the offeree makes a counteroffer, unless the offeror revives the original offer Mirror image rule: common law; if the acceptance deviates in the slightest from the terms of the offer, the acceptance constitutes a counteroffer and rejection Last shot rule: if the parties perform without an express contract, the last document governs

An innkeeper, who had no previous experience in the motel or commercial laundry business and who knew nothing about the trade usages of either business, bought a motel and signed an agreement with a laundry company for the motel's laundry services. The agreement was for a term of one year and provided for "daily service at $500 a week." From their conversations during negotiation, the laundry company owner knew that the innkeeper expected laundry services seven days a week. When the laundry company refused to pick up the motel's laundry on two successive Sundays and indicated that it would never do so, the innkeeper canceled the agreement. The laundry company sued the innkeeper for breach of contract. At trial, clear evidence was introduced to show that in the commercial laundry business "daily service" did not include service on Sundays. Is the laundry company likely to succeed in its action? •(A) No, because the laundry company knew the meaning the innkeeper attached to "daily service," and therefore the innkeeper's meaning will control. •(B) No, because the parties attached materially different meanings to "daily service," and therefore no contract was formed. •(C) Yes, because the parol evidence rule will not permit the innkeeper to prove the meaning he attached to "daily service." •(D) Yes, because trade usage will control the interpretation of "daily service."

Answer A

Becky, age 92, has only $25,000 to her name and is fearful that she will outlive her financial resources. On August 10, 2012, Becky explains her concern to her wealthy grand-nephew Logan. On that same day, by signed writing, Logan makes her this promise: "When you exhaust the 25,000 you now have, I will provide you, as a gift, any amount of money you request, up to a maximum of $50,000 per year for the remainder of your life." In September 2012, Becky's grandson Thane asks Becky for $25,000. Unable to resist, and believing that she can turn to Logan for any money she may need, Becky gives Thane the $25,000 in her bank account—all that she has in the world. Becky then contacts Logan. Explaining what she has done, she asks him for $25,000. Logan responds, "I did not make my promise to you so that you could give the money away to Thane. I'm not going to keep the promise." To what extent does the doctrine of promissory estoppel require that Logan keep his promise of August 10, 2012? A. Not at all, because Becky did not reasonably rely on it. B. To the extent of $25,000, because that is the extent to which Becky relied on it. C. To the extent of $25,000, because that is less than amount he promised to pay per year. D. Fully, because Becky was reasonable in believing that Logan would honor his promise.

Answer A

John and Mary agreed on the telephone that John would buy Mary's Ted Williams autographed baseball for $400. Mary later told him, "The joke's on you. I got a better offer for the ball, and all I had with you was an oral contract, and that doesn't count." Is there an enforceable contract between Mary and John? A. Yes, because oral agreements for the sale of goods for less than $500 are enforceable. B. Yes, because the UCC does not apply to this agreement because John and Mary are not merchants. C. Yes, because Mary admitted making the agreement. D. No, because there is no writing signed by Mary.

Answer A

S offers in writing to sell Blackacre, a parcel of land, to B for $100k. The writing provides that the offer will remain open until 11:00am. on June 7. At 11:15 on June 7, B arrives at S's house and states I'll pay $100k for Blackacre. In response, S states, "I'm sorry the offer is revoked." S then slams the door in B's face. Is there mutual assent? •A. No, because S's offer had expired. •B. No, because B rejected S's offer with a counteroffer. •C. Yes, because the June 7 deadline was not reasonable considering that the subject of the offer was real estate. •D. Yes, because B accepted S's offer before S revoked.

Answer A

•. A mechanic agreed in writing to make repairs to a landscaper's truck for $12,000. The mechanic properly made the repairs, but when the landscaper tendered payment, the mechanic refused to deliver the truck unless the landscaper promised to pay an additional $2,000. The customary charge for such work was $14,000. Because the landscaper needed the truck immediately to fulfill existing contractual obligations, and because no rental trucks of the same type were available, the landscaper promised in writing to pay the mechanic an additional $2,000. The mechanic then delivered the truck. Will the mechanic be able to enforce the landscaper's promise to pay the additional $2,000? •A. No, because the landscaper had no reasonable alternative but to yield to the mechanic's wrongful threat. •B. No, because the mechanic exerted undue influence over the landscaper with respect to the modification. •C. Yes, because the landscaper could have obtained possession of the truck through legal action rather than by agreeing to the increased payment. •D. Yes, because the modified contract price did not exceed a reasonable price.

Answer A

•. On July 18, Snowco, a shovel manufacturer, received an order for the purchase of 500 shovels from Acme, Inc., a wholesaler. Acme had mailed the purchase order on July 15. The order required shipment of the shovels no earlier than September 15 and no later than October 15. Typed conspicuously across the front of the order form was the following: "Acme, Inc. reserves the right to cancel this order at any time before September 1." Snowco's mailed response, saying, "We accept your order," was received by Acme on July 21. As of July 22, which of the following is an accurate statement as to whether a contract was formed? •A. No contract was formed because of Acme's reservation of the right to cancel. •B. No contract was formed because Acme's order was only a revocable offer. •C. A contract was formed, but prior to September 1 it was terminable at the will of either party. •D. A contract was formed, but prior to September 1 it was an option contract terminable at will of Acme.

Answer A

•. In order to raise revenue, a city required home-repair contractors who performed work within the city limits to pay a licensing fee to a city agency. A contractor who was unaware of the fee requirement agreed to perform home repairs for a city resident. After the contractor completed the work, the resident discovered that the contractor had not paid the licensing fee, and she refused to pay for the repairs, which were otherwise satisfactory. In the contractor sues the resident for breach of contract, how is the court likely to rule? •A. Although the contract violates the law and is void, the court will require the homeowner to pay the contractor the reasonable value of the work accepted. •B. Although the contract violates the law, the court will find that public policy does not bar enforcement of the contract, because the purpose of the fee is merely to raise revenue. •C. Because the contract violates the law and is void, the court will not enforce it. •D. Because the purpose of the fee is merely to raise revenue, the court will find that the contract does not violate the law but will allow the contractor to recover his costs only.

Answer B

•In a writing signed by both parties on December 1, K agreed to buy from S a gasoline engine for $1,000, delivery to be made on the following February 1. Through a secretarial error, the writing called for delivery on March 1, but neither party noticed the error until February 1. Before signing the agreement, K and S orally agreed that the contract of sale would be effective only if K should notify S in writing no later than January 2 that K had arranged to resell the engine to a third party. Otherwise, they orally agreed, "There is no deal." On December 15, K entered into a contract with T to resell the engine to T at a profit. K did not give S notice of the resale until January 25, and S received it by mail on January 26. Meantime, the value of the engine had unexpectedly increased about 75% since December 1, and S renounced the agreement. If K sues S on February 2 for breach of contract, which of the following is S's best defense? •(1) The secretarial error in the written delivery-term was a mutual mistake concerning a basic fact, and the agreement is voidable by either party. •(2) K's not giving written notice by January 2 of his resale was a failure of a condition precedent to the existence of a contract. •(3) In view of the unexpected 75% increase in value of the engine after December 1, S's performance is excused by the doctrine of commercial frustration. •(4) The agreement, if any, is unenforceable because a material term is not included in the writing.

Answer B/2

On August 1, Geriatrics, inc. operating a lifetime care home for elderly, admitted Ohlster who was 84 years old for a trial period of two months. On September 25, Ohlster and Geriatrics entered into a written lifetime care contract with an effective commencement date of October 1. The full contract price was $20,000 which as required by the terms of the contract, Ohlster prepaid to Geriatrics on September 25. Ohlster died of a heart attack on October 2. In a restituionary action, can the administrator of Ohlster's estate, a surviving sister, recover on behalf of the estate either all or part of the $20,000 paid to Geriatrics on September 25? A. Yes, because Geriatrics would otherwise be unjustly enriched at Ohlster's expense B. Yes, under the doctrine of frustration of purpose c. No, because Ohlster's life span and duration of Geriatrics' commitment to him was a risk assumed by both parties D. no, but only if Geriatrics can show that between septemeber 25 and Ohlster's death it rejected, because of its commitment to Ohlster, an application for lifetime care from another elderly person.

Answer C

•For an agreed price of $20 million, Bildko, Inc., contracted with Venture to design and build on Venture's commercial plot a 15-story office building. In excavating for the foundation and underground utilities, Bildko encountered a massive layer of granite at a depth of 15 feet. By reasonable safety criteria, the building's foundation required a minimum excavation of 25 feet. When the contract was made, neither Venture nor Bildko was aware of the subsurface granite, for the presence of which neither party had hired a qualified expert to test. Claiming accurately that removal of enough granite to permit the construction as planned would cost him an additional $3 million and a probable net loss on the contract of $2 million, Bildko refused to proceed with the work unless Venture would promise to pay an additional $2.5 million for the completed building. If Venture refuses and sues Bildko for breach of contract, which of the following will the court probably decide? •A. Bildko is excused under the modern doctrine of supervening impossibility, which includes severe impracticability. •B. Bildko is excused, because the contract is voidable on account of the parties' mutual mistake concerning an essential underlying fact. •C. Venture prevails, because Bildko assumed the risk of encountering subsurface granite that was unknown to Venture. •D. Venture prevails, unless subsurface granite was previously unknown anywhere in the vicinity of Venture's construction site.

Answer C

•. Dumont, a real estate developer, was trying to purchase land on which he intended to build a large commercial development. Perkins, an elderly widow, had rejected all of Dumont's offers to buy her ancestral home, where she had lived all her life and which was located in the middle of Dumont's planned development. Finally, Dumont offered her $250,000. He told her that it was his last offer and that if she rejected it, state law authorized him to have her property condemned. Perkins then consulted her nephew, a lawyer, who researched the question and advised her that Dumont had no power of condemnation under state law. Perkins had been badly frightened by Dumont's threat, and was outraged when she learned that Dumont had lied to her. If Perkins asserts a claim based on misrepresentation against Dumont, will she prevail? •A. Yes, if Dumont knew he had no legal power of condemnation. •B. Yes, if Dumont tried to take unfair advantage of a gross difference between himself and Perkins in commercial knowledge and experience. •C. No, if Dumont's offer of $250,000 equaled or exceeded the market value of Perkins's property. •D. No, because Perkins suffered no pecuniary loss.

Answer D

•Shortly after a series of burglaries took place within a city, the city council approved the offering of a $25,000 reward for the arrest and conviction of the perpetrator of the burglaries. Information concerning the reward was published in the local newspaper. In which of the following ways could the city's reward offer be effectively accepted? •A. Only by an offeree's return promise to make the arrest and assist in the successful conviction of a burglar within the scope of the offer. •B. Only by an offeree's return promise to make a reasonable effort to bring about the arrest and conviction of a burglar within the scope of the offer. •C. By an offeree's communication of assent through the same medium (local newspaper) used by the city in making its offer. •D. By an offeree's supplying information leading to arrest and conviction of a burglar within the scope of the offer.

Answer D

A man offered to sell a piano to a female aquaintance for 400$. The woman had been to the man's house and knew that he owned a stein berg piano, so she accepted. unbeknownst to the woman, the man also owned a hairwin piano, and that was the piano that he intended to sell, although he was aware that the woman had only seen he stein berg. If the woman sues the man to obtain the stein berg. who will prevail? •A. The woman, because the man knew of the ambiguity. • B. The woman, because that was her objective intent. • C. The man, because there was a mutual mistake. • D. The man, because he subjectively intended to sell the Hairwin instead of the Steinberg.

Answer: A

Chad owned a house and lot that he orally agreed to sell to Lynn for $230k. Lynn paid chad a deposit of $25k after several weeks of unsuccessfully trying to find a lender who would loan her the balance of the money to buy the house, Lynn finally called Chad and said: "the deal is off. Send my deposit back." chad refused to do so. based upon the foregoing, if Lynn sued chad to recover her 35k deposit, the most likely result will be that: •A. Chad must repay the $25,000. •B. Lynn will not be able to compel Chad to return the $25,000 since there was an enforceable agreement. •C. Lynn will not be able to compel the return of the $25,000 since the agreement is enforceable because of part performance (i.e., her down payment to Chad). •D. Lynn cannot compel the return of the $25,000, butcan recover her actual damages.

Answer: A

Ozzie owned and occupied Blackacre, which was a tract of land improved with a one-family house. His friend, Victor, orally offered Ozzie $50,000 for Blackacre, the fair market value, and Ozzie accepted. Because they were friends, they saw no need for attorneys or written contracts and shook hands on the deal. Victor paid Ozzie $5,000 down in cash and agreed to pay the balance of $45,000 at an agreed closing time and place. Before the closing, Victor inherited another home and asked Ozzie to return his $5,000. Ozzie refused, and, at the time set for the closing, Ozzie tendered a good deed to Victor and declared his intention to vacate Blackacre the next day. Ozzie demanded that Victor complete the purchase. Victor refused. The fair market value of Blackacre has remained $50,000. In an appropriate action brought by Ozzie against Victor for specific performance, if Ozzie loses, the most likely reason will be that A. the agreement was oral. B. keeping the $5,000 is Ozzie's exclusive remedy. C. Victor had a valid reason for not closing. D. Ozzie remained in possession on the day set for the closing.

Answer: A

Rob, an auto retailer, had an adult daughter, Betsy, who needed a car in her employment but had only $3,000 with which to buy one. Rob wrote to her, "Give me $3,000 and I'll give you the car on the lot that we have been using as a demonstrator." Betsy thanked her father and paid him the $3,000. As both Rob and Betsy knew, the demonstrator was reasonably worth $10,000. After Betsy had paid the $3,000, but before the car had been delivered to her, one of Rob's sales staff sold and delivered the same car to a customer for $10,000. Neither the salesperson nor the customer was aware of the transaction between Rob and Betsy. Does Betsy have an action for breach of contract? A. Yes, because Rob's promise was supported by bargained-for consideration. B. Yes, because Rob's promise was supported by the moral obligation a father owes his child as to the necessities of life. C. No, because the payment of $3,000 was inadequate consideration to support Rob's promise. D. No, because the salesperson's delivery of the car to the customer revoked Rob's offer.

Answer: A

A man fed a lost cat and returned it to its owner. She was so happy to see her cat that she promised to leave the man money in her will. When the cat owner died a few days later, the man visited the owner's daughter demanding his share of the estate. All of the cat owner's bank accounts had been join tenancy with her daughter. The daughter reluctantly signed a document stating that she would pay the man $500 in exchange for his agreement not to sue her mother's estate. She later had second thoughts and refused to pay the man on the ground that there was no consideration for the agreement. Besides the consideration stated in the daughter's written instrument, what other fact would strengthen the man's claim? •A. He never would have fed and returned the cat had he known that he would not receive any payment for his efforts. •B. He believed he could sue the cat owner's estate. •C. The majority of the funds in the cat owner's bank account were royalties from a series of television commercials starring the cat. •D. The cat owner's promise to him was in writing.

Answer: B

Dominique obtained a bid of $10,000 to tear down her old building and another bid of $90k to replace it with a new structure in which she planned to operate a sporting goods store. Having only limited cash available, D asked hard cash for a $100k loan. After reviewing the plans for the project, hard cash in a signed writing promised to lend D $100k secured by a mortfafe on the property and repayable over ten years in equal monthly installments at 10% annual interest. D promptly accepted the demolition bid and old building was removed, but hard cash thereafter refused to make the loan. Despite diligent efforts, D was unable to obtain a loan from any other source. Does D have a cause of action against Hardcash? •(A) Yes, because by having the building demolished, she accepted Hardcash's offer to make the loan. •(B) Yes, because her reliance on Hardcash's promise was substantial, reasonable, and foreseeable. •(C) No, because there was no bargained-for exchange of consideration for Hardcash's promise to make the loan. •(D) No, because Dominique's inability to obtain a loan from any other source demonstrated that the project lacked the financial soundness that was a constructive condition to Hardcash's performance.

Answer: B

In answer to a radio advertisement, a teenager two months shy of his 18th birthday contracted to buy a late model car form a car dealership. The agreement required a $1.5k down payment with the remainder of the $7.2k price to be paid in monthly installments to a local finance company. The teenager's first 8 payments were made regularly until his driver's license was suspended. He then informed the company that no further payments would be forthcoming. The finance company sued for the remaining payments. The age of majority in the teenager's state is 18 years. Would the teenager be liable for the balance of the payments? A. yes, because the car dealership was liable on the contract from the outset, not withstanding the teenager's minority B. Yes, because he kept the car for six month after reaching the age of majoirty C. no, recuasse he was a minor at the time of contracting and the contract is voided by him D. No, becuase he informed the finance company in a timely manner after his driver's license was suspended

Answer: B

Landowner was land rich by inheritance but money poor, having suffered severe losses on bad investments, but still owned several thousand acres of unencumbered timberland. He had a large family, and his normal, fixed personal expenses were high. Pressed for cash, he advertised a proposed sale of standing timber on a choice 2,000 acre tract. The only response was an offer by a logger, the owner of a large, integrated construction enterprise, after inspection of the advertised tract. Logger offered to buy, sever, and remove the standing timber form the advertised tract at a cash price of 70% lower than the regionally prevailing price for comparable timber rights. Landholder, by then in desperate financial straits and knowing little about timber values, signed and delivered to Logger a letter accepting the offer. If before Logger commences performance, landholder's investment fortunes suddenly improve and he wishes to get out of the timber deal with logger, which of the following legal concepts affords his best prospects of effective cancellation?

Answer: B

employee age 60 who had no plans for early retirement, had worked for employer for 20 years as a managerial employee at will when he had a conversation with employers president, about employees post retirement goal of extensive travel around the US. A month later, Presidet handed employee, a written signed resolution of employers board of directors stating that when and if employee should decide to retire, at his option, the company, in recognition of his past service, would pay him a $4,000 per month lifetime pension. The company had no regularized retirement plan for at will employees. Shortly thereafter, employee retired and immediately bought a 50k vehicle for his planned travels. After receiving the promised $4,000 monthly pension from employer advising employee now unemployable elsewhere, received a letter from employer advising him that the pension would cease immediately becuaseof recessionary budget restraints affecting in varying degrees all managerial salaries and retirement pensions. In a suit against employer for damages, employee will probably: •A. win, because he retired from the company as bargained-for-consideration for the Board's promise to him of a lifetime pension. •B. win, because he timed his decision to retire and to buy the recreational vehicle in reasonable reliance on the Board's promise to him of a lifetime pension. •C. lose, because the Board's promise to him of a lifetime pension was an unenforceable gift promise. •D. lose, because he had been an employee-at-will throughout his active service with the company.

Answer: B

•A seller and a buyer have dealt with each other in hundreds of separate grain contracts over the last five years. In performing each contract, the seller delivered the grain to the buyer and, upon delivery, the buyer signed an invoice that showed an agreed-upon price for that delivery. Each invoice was silent in regard to any discount from the price for prompt payment. The custom of the grain trade is to allow a 2% discount from the invoice price for payment within 10 days of delivery. In all of their prior transactions and without objection from the seller, the buyer took 15 days to pay and deducted 5% from the invoice price. The seller and the buyer recently entered into a contract for a single delivery of wheat at a price of $300,000. The same delivery procedure and invoice were used for this contract as had been used previously. The seller delivered the wheat and the buyer then signed the invoice. On the third day after delivery, the buyer received the following note from the seller: "Payment in full in accordance with signed invoice is due immediately. No discounts permitted. /s/Seller." Which of the following statements concerning these facts is most accurate? •A. The custom of the trade controls, and the buyer is entitled to take a 2% discount if he pays within 10 days. •B. The parties' course of dealing controls, and the buyer is entitled to take a 5% discount if he pays within 15 days. •C. The seller's retraction of his prior waiver controls, and the buyer is entitled to no discount. •D. The written contract controls, and the buyer is entitled to no discount because of the parol evidence rule.

Answer: B

•Five years ago, Sally acquired Blackacre, improved with a 15-year-old dwelling. This year Sally listed Blackacre for sale with Bill, a licensed real estate broker. Sally informed Bill of several defects in the house that were not readily discoverable by a reasonable inspection, including a leaky basement, an inadequate water supply, and a roof that leaked. Paul responded to Bill's advertisement, was taken by Bill to view Blackacre, and decided to buy it. Bill saw to it that the contract specified the property to be "as is" but neither Bill nor Sally pointed out the defects to Paul, who did not ask about the condition of the dwelling. After closing and taking possession, Paul discovered the defects, had them repaired, and demanded that Sally reimburse him for the cost of the repairs. Sally refused and Paul brought an appropriate action against Sally for damages. If Sally wins, it will be because •(A) Sally fulfilled the duty to disclose defects by disclosure to Bill. •(B) The contract's "as is" provision controls the rights of the parties. •(C) Bill became the agent of both Paul and Sally and thus knowledge of the defects was imputed to Paul. •(D) The seller of a used dwelling that has been viewed by the buyer has no responsibility toward the buyer.

Answer: B

A daughter was appointed guardian of her elderly father following an adjudication of his mental incompetence. The father had experienced periods of dementia during which he did not fully understand what he was doing. The father later contracted to purchase an automobile at a fair price from a seller who was unaware of the gaurdianship. At the time of the purchase of the transaction. What is the legal status of the transaction? A. the contract is enforceable, because a reasonable person in the situation of the seller would have thought that the father had the capacity to make the contract B. the contract is enforceable becuase it was made on fair terms and the seller had no knowledge of the father's gaurdianship C. the contract is void, because the father was under the gaurdianship at the time it was made D. the contract is voidable at the option of the father

Answer: C

•2. Purvis purchased a used car from Daley, a used car dealer. Knowing that they were false, Daley made the following statements to Purvis prior to the sale: -Statement 1. This car has never been involved in an accident. -Statement 2. This car gets 25 miles to the gallon on the open highway. -Statement 3. This is as smooth-riding a car as you can get. •If Purvis seeks to rescind the contract based on fraud, which of the false statements made by Daley would support Purvis' claim? •(A) Statement 1 only. •(B) Statement 2 only. •(C) Statements 1 and 2 only. •(D) Statements 2 and 3 only.

Answer: C

•Breeder bought a two-month-old registered boar at auction from Pigstyle for $800. No express warranty was made. Fifteen months later, tests by experts proved conclusively that the boar had been born incurably sterile. If this had been known at the time of the sale, the boar would have been worth no more than $100. In an action by Breeder against Pigstyle to avoid the contract and recover the price paid, the parties stipulate that, as both were and had been aware, the minimum age at which the fertility of a boar can be determined is about 12 months. Which of the following will the court probably decide? •A. Breeder wins, because the parties were mutually mistaken as to the boar's fertility when they made the agreement. •B. Breeder wins, because Pigstyle impliedly warranted that the boar was fit for breeding. •C. Pigstyle wins, because Breeder assumed the risk of the boar's sterility. •D. Pigstyle wins, because any mistake involved was unilateral, not mutual.

Answer: C

•O orally agrees to sell Blackacre to B. B pays O the full price, takes possession, and begins to plant crops on Blackacre. O, however, repudiates the agreement. If B sues for specific performance, B's best argument would be: •A. the Statute of Frauds. •B. the doctrine of promissory estoppel. •C. the doctrine of part performance. D. the doctrine of equitable conversion.

Answer: C

•A homeowner orally contracted to buy some custom made parts that the homeowner will use to build a retaining wall in the homeowner's back yard. The contract price for these materials was $2,000. The homeowner refused to take delivery even though the goods were conforming. The seller is unable to sell these parts to anyone else because they were individually designed for the homeowner. In a lawsuit against the homeowner by the seller of the parts: •A. The UCC would not apply because these goods will become real property when installed by the homeowner. •B. This contract is not enforceable because the price is more than $500. •C. This contract is not enforceable, unless the homeowner is a merchant in the materials purchased. •D. This contract is fully enforceable even though it is not in writing.

Answer: D

•4. The mother of a son and a daughter was dying. The daughter visited her mother in a hospice facility and said, "You know that I have always been the good child, and my brother has always been the bad child. Even so, you have left your property in the will to us fifty-fifty. But it would be really nice if you would sell me the family home for $100,000." "I don't know," said the mother. "It is worth a lot more than that—at least $250,000." "That is true," said the daughter. "But I have always been good and visited you, and my brother has never visited you, so that ought to be worth something. And besides, if you won't sell me the house for that price, maybe I won't visit you anymore, either." "Oh, I wouldn't want that," said the mother, and she signed a contract selling the house to her daughter for $100,000. Shortly thereafter, the mother died. When her son found out that the house had been sold and was not part of his mother's estate, he sued to have the contract avoided on behalf of the mother. On what ground would the contract most likely be avoided? •(A) Duress. •(B) Inadequate consideration. •(C) Mistake. •(D) Undue influence.

Answer: D.

A young man proposed to his girlfriend, but she was reluctant becuase of his meager income and lack of job potential. The young man told his father about her reluctance. the father told the girlfriend that id she married the son, he would support them for 6 months and send the son to a six month computer technology training school. This was sufficient to dispel her reservations and the two were married soon thereafter. When they returned from their honeymoon, the father refused to go through with his offer. Although the girlfriend is happy in her marriage, she sued the father for damages. If the father prevails it will be because: A. the father's promise was not supported by valid consideration B. the contract is against public policy C. the contract was oral D. the girlfriend is happy and therefore has incurred no detriment

Answer: c

•8. Assume the same facts as the last question. For this question only, assume that Dominique has a cause of action against Hardcash. If she sues him for monetary relief, what is the probable measure of her recovery? •(A) Expectancy damages, measured by the difference between the value of the new building and the old building, less the amount of the proposed loan ($100,000). •(B) Expectancy damages, measured by the estimated profits from operating the proposed sporting goods store for ten years, less the cost of repaying a $100,000 loan at 10% interest over ten years. •(C) Reliance damages, measured by the $10,000 expense of removing the old building, adjusted by the decrease or increase in the market value of Dominique's land immediately thereafter. •(D) Nominal damages only, because both expectancy and reliance damages are speculative, and there is no legal or equitable basis for awarding restitution.

Answer:C

•. An insurer offered a plan to cover an insured's catastrophic illnesses for the remainder of the insured's life in exchange for a large one-time payment at the inception of coverage. Because the program was experimental, the insurer would accept only a fixed number of applications during the enrollment period. A recent retiree in good health was one of the applicants accepted, and he enrolled in the program. He paid the one-time premium of $30,000 a few days before coverage began. The day after his coverage started, he was struck by a bus and killed. The executor of the retiree's estate reviewed the policy and immediately notified the bank to stop payment on it. The insurer then filed suit against the retiree's estate. Will the court compel the estate to pay the premium to the insurer? •A. Yes, because the insurer necessarily declined to take another applicant during the enrollment period because of the retiree's promise to buy the policy. •B. Yes, because the risk of the timing of the retiree's death was assumed by both parties and built into the cost of the contract. •C. No, because the purpose of the contract between the retiree and the insurer had been frustrated. •D. No, because it is unconscionable for the insurer to have charged the retiree so much for so little value received.

AnswerB

Under the terms of a written contract, Karp agreed to construct for Manor a garage for •$10,000. Nothing was said in the parties' negotiations or in the contract about progress payments during the course of the work. After completing 25% of the garage strictly according to Manor's specifications, Karp demanded payment of $2,000 as a "reasonable progress payment." Manor refused, and Karp abandoned the job. If each party sues the other for breach of contract, which of the following will the court decide? •A) Both parties are in breach, and each is entitled to damages, if any, from the other. •(B) Only Karp is in breach and liable for Manor's damages, if any. •(C) Only Manor is in breach and liable for Karp's damages, if any. •(D) Both parties took reasonable positions and neither is in breach.

B

•. A homeowner and a builder entered into a written contract to add a spare room on to the homeowner's home at a cost of $13,000. The contract contained a clause stating that the builder will not begin construction without prior approval of the plans by the homeowner's financial advisor. The builder submitted his designs to both the homeowner and the financial advisor. The homeowner liked the plans, but the financial advisor did not and withheld his approval. The builder asked the homeowner whether she wanted him to submit new designs. The homeowner told the builder orally, "No! Your designs are great, so go right ahead and construct the new room." The builder constructed the new room. The homeowner now refuses to pay the builder, citing the clause requiring approval by the financial advisor. If the builder sues the homeowner, the builder will recover: •(A) The full contract price, because the financial advisor's approval was not a condition precedent for the contract to take effect. •(B) The full contract price, because the builder detrimentally relied on the homeowner's waiver of the condition precedent. •(C) The reasonable value of the builder's services and materials, because otherwise the homeowner would be unjustly enriched. •(D) Nothing, because the homeowner's oral statement will be excluded by the parol evidence rule.

B

•. A park board in a large suburb announced that it was accepting bids for renovation work on its recreation center. A builder advertised for sub-bids for the electrical work, and a local electrician submitted to the builder by electronic bidding service a sub-bid of $130,000. However, due to the bidding service's negligence, the sub-bid that the builder received from the electrician read $30,000 instead of $130,000. Because this was the lowest sub-bid that the builder received for the electrical work, and $60,000 less than the next lowest sub-bid, the builder awarded the subcontract to the electrician. Based in part on the electrician's sub-bid, the builder came up with a bid for the job that beat out all of the competition and thus won the job. The electrician's best argument to successfully refuse to perform the resulting contract is: •A. The contract is voidable due to mutual mistake. •B. The great difference between the $30,000 figure and the next lowest bid should have alerted the builder to the existence of a mistake in the sub-bid. •C. The electrician was not responsible for the negligence of the bidding service. •D. The builder's own negligence in not checking out all sub-bids precludes enforcement of the contract.

B

•. Buyer and Seller deal in electrical equipment. In its catalog, Seller lists the prices for some of its products, but not for #4 conducting wire. On July 11, Buyer sends Seller this signed written message: "We wish immediately to have 4,000 feet of your #4 conducting wire. We do not see the price listed in your catalog, so we propose that the price be agreed on later. Please ship." On July 12, Seller responds with this signed writing: "Thank you. We will ship." Have Buyer and Seller formed a contract? •A. Yes, because where the goods are bought and sold between merchants, price is not a material term. •B. Yes, because notwithstanding the open price term, their interaction reflects finality of agreement. •C. No, because parties do not form a contract if they agree that a material term will be subject to future agreement. •D. No, because a seller who declines to publish its price shows an intention not to sell.

B

•. Fixtures, Inc., in a signed writing, contracted with Apartments for the sale to Apartments of 50 identical sets of specified bathroom fixtures, 25 sets to be delivered on March 1, and the remaining 25 sets on April 1. The agreement did not specify the place of delivery, or the time or place of payment. Which of the following statements is correct? •A. Fixtures must tender 25 sets to Apartments at Apartments place of business on March 1, but does not have to turn them over to Apartments until Apartments pays the contract price for the 25 sets. •B. Fixtures has no duty to deliver the 25 sets on March 1 at Fixtures place of business unless Apartments tenders the contract price for the 25 sets on that date. •C. Fixtures must deliver 25 sets on March 1, and Apartments must pay the contract price for the 25 sets within a reasonable time after their delivery. D.Fixtures must deliver 25 sets on March 1, butApartmentspayment is due only upon the delivery of all 50 sets.

B

•. P and D enter into a written contract, the essential part of which reads as follows: "P to paint D's portrait and complete it on or before December 15. Portrait guaranteed to be fully satisfactory to D." The portrait was completed and delivered to D on December 15. Although the portrait pleased D's family and friends, D refused to accept the portrait, stating "it makes me look old and mean." In an action by P against D, which of the following would be D's best defense? •(A) The portrait, objectively viewed, was not satisfactory. •(B) The portrait, subjectively viewed, was not satisfactory. •(C) The portrait would expose D to ridicule. •(D) At least some reasonably prudent artists would reject the portrait.

B

•. Stirrup, a rancher, and Equinox, a fancier of horses, signed the following writing: "For​ $5,000, Stirrup will sell to Equinox a gray​ horse that Equinox may choose from among​ the grays on Stirrup's ranch."​ Equinox refused to accept delivery of a gray​ horse timely tendered by Stirrup or to choose​ among those remaining, on the ground that​ during their negotiations Stirrup had orally​ agreed to include a saddle, worth $100, and​ also to give Equinox the option to choose a​ gray or a brown horse. Equinox insisted on​ one of Stirrup's brown horses, but Stirrup​ refused to part with any of his browns or with​ the saddle as demanded by Equinox.​ If Equinox sues Stirrup for damages and seeks​ to introduce evidence of the alleged oral​ agreements, the court probably will​ •(A) admit the evidence as to both the saddle​ and the option to choose a brown horse.​ •(B) admit the evidence as to the saddle but​ not the option to choose a brown horse.​ •(C) admit the evidence as to the option to​ choose a brown horse but not the​ promise to include the saddle.​ (D) not admit any of the evidence.

B

•3. Green contracted in a signed writing to sell Greenacre, a 500-acre tract of farmland, to Farmer. The contract provided for exchange of the deed and purchase price of $500,000 in cash on January 15. Possession was to be given to Farmer on the same date. On January 15, Green notified Farmer that because the tenant on Greenacre wrongfully refused to quit the premises until January 30, Green would be unable to deliver possession of Greenacre until then, but he assured Farmer that he would tender the deed and possession on that date. When Green tendered the deed and possession on January 30, Farmer refused to accept either, and refused to pay the $500,000. Throughout the month of January, the market value of Greenacre was $510,000, and its fair monthly rental value was $5,000. Will Green probably succeed in an action against Farmer for specific performance? •(A) Yes, because the court will excuse the delay in tender on the ground that there was a temporary impossibility caused by the tenant's holding over. •(B) Yes, because time is ordinarily not of the essence in a land-sale contract. •(C) No, because Green breached by failing to tender the deed and possession on January 15. (D) No, because Green's remedy at law for monetary relief is adequate

B

•A builder learned of a bid being let by the local school board for a new high school. Anxious to get the job, the builder immediately advertised for sub-bids. The lowest sub-bid came from a local contractor. It was for $180,000, which was $10,000 less than the next lowest sub-bid. The builder submitted its general bid to the school board after computing it based, in part, on the lowest sub-bid. The builder's general bid was $10,000 lower than the school board had originally anticipated. Pleasantly surprised by the low bid, the school board accepted. After the acceptance, the contractor who had submitted the lowest sub-bid informed the builder that there had been a mistake in computing the sub-bid. The correct sub-bid was $200,000. The builder had been unaware of this mistake. In a suit for rescission of its contract with the builder, the contractor should: •A) Not prevail, because the mistake in the figures was made before the bid was accepted. •B) Not prevail, because nothing indicates that the builder actually knew or should have known of the mistake involved. •C) Prevail, because the mistake was part of the basis of the bargain. •D) Prevail, because this was a unilateral, not a bilateral, mistake.

B

•A buyer purchases some equipment that costs $20,000. As part of the same contract, the seller, a parts retailer, agrees to install this equipment for an additional $2,000. After the buyer begins using the equipment, problems develop. Some of the problems relate to how the equipment was installed, while others relate to the equipment itself. Which of the following is probably true in most states? •A. The sales portion of the contract is covered by the UCC, but the installation services are covered under common law. •B. The entire contract is covered under the UCC. •C. The entire contract is covered under common law. •D. The entire contract is covered under either the UCC or the entire contract is covered under common law; which one covers it depends on whether more of the problems relate to the equipment itself or to the installation.

B

•A company agreed to employ a businesswoman for two years as a vice president at a salary of $250,000 per year. After six months, the company, without cause, fired the businesswoman. Which of the following statements best describes the businesswoman's rights after being fired? •(A) She can recover the promised salary for the remainder of the two years if she remains ready to work. •(B) She can recover the promised salary for the remainder of the two years if no comparable job is reasonably available and she does not take another job. •(C) She can recover the promised salary for the remainder of the two years regardless of her efforts to mitigate damages. •(D) She can get specific performance of her right to serve as a vice president of the company for two years. • •If the business woman spent $300 traveling to an interview for a comparable job but was not chosen for that job, could she recover the $300 in addition to her other damages? -Yes, as incidental damages.

B

•A consumer goes to a jewelry store and points out to the clerk a particular watch on display that he wishes to purchase. The clerk goes to the back of the store and emerges with a watch that he briefly shows to the consumer before wrapping it. The consumer buys the watch. When he gets home, the consumer discovers that the watch he purchased differs from the one that he pointed out in the display. Does the consumer have a claim for breach of warranty? •A. Yes, because an express warranty was created by affirmation of fact or promise. •B. Yes, because an express warranty was created by sample or model. •C. No, because he had an opportunity to inspect the goods before purchase. •D. No, because there was no language of warranty.

B

•A large dairy regularly supplied milk to local grocery stores. A large grocery store located in the same city as the dairy had not previously done business with the dairy. It sent a written order to the dairy for 50 cases of homogenized whole milk at the wholesale market price of $10 per case for prompt shipment. The warehouse clerk at the dairy facility negligently shipped 50 cases of low-fat milk instead of the grade that was ordered. When the milk is delivered at the grocery store by a common carrier, which of the following is the most appropriate statement of the parties' rights and duties? •A. The store must accept the delivery, even though it is nonconforming, because a contract was formed when the dairy shipped it. •B. A contract was formed but breached by the dairy when it shipped nonconforming goods, which the store has no obligation to accept. •C. Because the goods are nonconforming, the store can treat the delivery as a counteroffer and refuse acceptance. •D. The dairy's shipment of the milk is a counteroffer, but the store is under an obligation to reship it to the dairy at the dairy's expense if it decides not to accept the delivery.

B

•A law professor sells a refrigerator that she owns to a student. After the purchase, Sears repossesses the refrigerator from the student because the law professor gave Sears a security interest in it. Assuming it was legal for Sears to do this, does the student have a claim against the professor for breach of warranty? •A. Trick question! Article 2 doesn't apply to this transaction. •B. Yes, the professor warranted that the goods were free from any security interest. •C. No. Because the professor owned the refrigerator, she transferred good title to the student. •D. No, because it is the responsibility of the buyer to check for security interests.

B

•A producer hired a violinist to play in an orchestra that was to leave on a 10-week tour. The violinist turned down another job opportunity in order to accept the producer's job offer. One week after the start of the tour, the violist was hospitalized with a bad back and was unable to perform. The producer hired another musician to take her part in the orchestra. Four days later, the violist recovered but the producer refused to allow her to rejoin the orchestra or to complete the tour. She then sued the producer for breach of contract. Which of the following is the violinist's best legal theory? •A. Her reliance on the job offered by the producer by declining another job opportunity created an estoppel against the producer. •B. Her failure to perform with the orchestra for four days was not a material failure so as to discharge the producer's duty to perform. •C. Her performance with the orchestra for the four-day period was physically impossible. •D. She was never told that an injury might jeopardize her continued employment with the orchestra.

B

•A producer of cherries offers his crop to a fruit wholesaler. The producer shows the wholesaler sample cherries from his crop and, in a written agreement, the wholesaler agrees to buy the entire cherry crop for a certain price. When the cherries arrive, most of them are less plump and less ripe than the ones the producer had shown to the wholesaler. Does the wholesaler have a claim for breach of warranty? •A. Yes, but only if the producer told the wholesaler the cherries would be plump and ripe. •B. Yes, because the sample created an express warranty that the goods would conform to that sample. •C. No, because no warranty was in writing. •D. No, because the wholesaler bore the risk that the goods did not conform to the sample.

B

•After negotiations, a painting contractor agreed with a homeowner to paint the exterior of her house for $20,000. The painter further agreed to use only top quality paint because the homeowner wanted the paint to last for many years. The painter filled out his standard form contract, but deliberately did not include a brand name for the paint that he would use. After telling the homeowner that the contract contained the terms agreed to, she signed without reading it. On completion of the job, the homeowner saw some empty paint cans in the painter's truck and realized that top quality paint had not been used. If the homeowner sues the paint contractor for misrepresentation, any statements made by the painter before the contract was signed: • A. Would be barred by the parol evidence rule. • B. Would not be barred by the parol evidence rule. • C. Would be admitted if the contract was proved to be an incomplete agreement of the parties. • D. Would not be admitted because they occurred prior to the signing of the contract.

B

•An engineer entered into a written contract with an owner to serve in the essential position of on-site supervisor for construction of an office building. The day after signing the contract, the engineer was injured while bicycling and was rendered physically incapable of performing as the on-site supervisor. The engineer offered to serve as an off-site consultant for the same pay as originally agreed to by the parties. Is the owner likely to prevail in an action against the engineer for damages resulting from his failure to perform under the contract? •(A) No, because the engineer offered a reasonable substitute by offering to serve as an off-site consultant. •(B) No, because the engineer's physical ability to perform as on-site supervisor was a basic assumption of the contract. •(C) Yes, because the engineer breached the contract by disappointing the owner's expectations. •(D) Yes, because the engineer's duty to perform was personal and absolute.

B

•At a country auction, P acquired an antique cabinet that he recognized as a "Morenci," an extremely rare and valuable collector's item. Unfortunately, P's cabinet had several coats of varnish and paint over the original finish. Its potential value could only be realized if these layers could be removed without damaging the original oil finish. A professional restorer of antique furniture recommended that P use Restorall to remove the paint and varnish from the cabinet. P obtained and read a sales brochure published by Restorall, Inc., which contained the following statement: "This product will renew all antique furniture. Will not damage original oil finishes." P purchased some Restorall and used it on his cabinet, being careful to follow the instructions exactly. Despite P's care, the original Morenci finish was irreparably damaged. When finally refinished, the cabinet was worth less than 20% of what it would have been worth if the Morenci finish had been preserved. If P sues Restorall, Inc. to recover the loss he has suffered as a result of the destruction of the Morenci finish, will P prevail? •A. Yes, unless no other removal technique would have preserved the Morenci finish. •B. Yes, if the loss would not have occurred had the statement in the brochure been true. •C. No, unless the product was defective when sold by Restorall, Inc. •D. No, if the product was not dangerous to persons.

B

•Bob thought Sam's home was beautiful and he often told Sam if he ever wanted to sell it he (Bob) would like to buy it. On June 10, Sam decided to sell his house to Bob. He drafted a real estate contract to sell the house to Bob, signed it and mailed it to Bob. The document had all necessary terms. Bob received the contract on June 12. He immediately signed it and mailed it back to Sam. Later that day, Bob had second thoughts and decided to cancel the agreement. He called Sam on June 13 and told him to ignore the signed contract because he had changed his mind. Sam received the signed contract the next day, June 14. Does Sam have an enforceable contract for the sale of his home under the broadly adopted view in the United States? •A. Yes, because Bob told Sam he was withdrawing his acceptance by telephone instead of by mail. •B. Yes, because Bob's act of mailing the signed document was an effective acceptance of Sam's offer. •C. No, because Bob effectively retracted his acceptance before Sam received it. •D. No, because the U.S. Postal Service was an agent for Bob, not for Sam, and under U.S. Postal Regulations Bob might have retrieved the document after mailing it back to Sam on June 12.

B

•Buyer, Inc. contracted in writing with Shareholder, who owned all of XYZ Corporation's outstanding stock, to purchase all of her stock at a specified price per share. At the time this contract was executed, Buyer's contracting officer said to Shareholder, "Of course, our commitment to buy is conditioned on our obtaining approval of the contract from Conglomerate, Ltd., our parent company." Shareholder replied, "Fine. No problem." Shareholder is willing and ready to consummate the sale of her stock to Buyer, but the latter refuses to perform on the ground (which is true) that Conglomerate has firmly refused to approve the contract. If Shareholder sues Buyer for breach of contract and seeks to exclude any evidence of the oral condition requiring Conglomerate's approval, the court will probably •A. admit the evidence as proof of a collateral agreement. •B. admit the evidence as proof of a condition to the existence of an enforceable obligation, and therefore not within the scope of the parol evidence rule. •C. exclude the evidence on the basis of a finding that the parties' written agreement was a complete integration of their contract. •D. exclude the evidence as contradicting the terms of the parties' written agreement, whether or not the writing was a complete integration of the contract.

B

•During negotiations between two friends over the sale of a boat, the seller tells the buyer that the engine was warranted to be free of defects for a period of two years. The parties finally sign a written agreement that contains no warranty terms. After 14 months, the engine develops problems because of a defect. A court finds that the writing is a partial integration and that the promise was actually made. Is the seller liable under the warranty? •A) Yes, because the gap fillers will supply an implied warranty. •B) Yes, because the oral promise does not contradict the terms of the writing. •C) No, because the writing was a final expression of their agreement as to the warranty. •D) No, because the buyer would have a claim even under the written warranty.

B

•Filmore purchased a Miracle color television set from Allison Appliances, an authorized dealer, for $499. The written contract contained the usual one-year warranty as to parts and labor as long as the set was returned to the manufacturer or one of its authorized dealers. The contract also contained an effective disclaimer of any express warranty protection, other than that which was included in the contract. It further provided that the contract represented the entire agreement and understanding of the parties. Filmore claims that during the bargaining process Surry, Allison's agent, orally promised to service the set at Filmore's residence if anything went wrong within the year. Allison has offered to repair the set if it is brought to the service department, but denies any liability under the alleged oral express agreement. Which of the following would be the best defense for Allison to rely upon in the event Filmore sues? •(A) The Statute of Frauds. •(B) The parol evidence rule. •(C) The fact that all warranty protection was disclaimed other than the express warranty contained in the contract. •(D) There was no consideration given for the oral agreement.

B

•Mural, a wallpaper hanger, sent Gennybelle, a general contractor, this telegram: "Will do all paperhanging on new Doctors' Building, per owner's specs, for $14,000 if you accept within reasonable time after main contract awarded." /s/ Mural. Three other competing hangers sent Gennybelle similar bids in the respective amounts of $18,000, $19,000, and $20,000. Gennybelle used Mural's $14,000 figure in preparing and submitting her own sealed bid on Doctors' Building. Before the bids were opened, Mural truthfully advised Gennybelle that the former's telegraphic sub-bid had been based on a $4,000 computational error and was therefore revoked. Shortly thereafter, Gennybelle was awarded the Doctors' Building construction contract and subsequently contracted with another paperhanger for a price of $18,000. Gennybelle now sues Mural to recover $4,000. Which of the following, if proved, would most strengthen Gennybelle's prospect of recovery? •A. After Mural's notice of revocation, Gennybelle made a reasonable effort to subcontract with another paperhanger at the lowest possible price. •B. Gennybelle had been required by the owner to submit a bid bond and could not have withdrawn or amended her bid on the main contract without forfeiting that bond. •C. Mural was negligent in erroneously calculating the amount of his sub-bid. •D. Gennybelle dealt with all of her subcontractors in good faith and without seeking to renegotiate (lower) the prices they had bid.

B

•On March 1, Mechanic contracted to repair Textiles' knitting machine and to complete the job by March 6. On March 2, Textiles contracted to manufacture and deliver specified cloth to Knitwear on March 15. Textiles knew that it would have to use the machine then under repair to perform this contract. Because the Knitwear order was for a rush job, Knitwear and Textiles included in their contract a liquidated damages clause, providing that Textiles would pay $5,000 for each day's delay in delivery after March 15. Mechanic was inexcusably five days late in repairing the machine, and, as a result, Textiles was five days late in delivering the cloth to Knitwear. Textiles paid $25,000 to Knitwear as liquidated damages and now sues Mechanic for $25,000. Both Mechanic and Textiles knew when making their contract on March 1 that under ordinary circumstances Textiles would sustain little or no damages of any kind as a result of a five-day delay in the machine repair. Assuming that the $5,000 liquidated damages clause in the Knitwear-Textiles contract is valid, which of the following arguments will serve as Mechanic's best defense to Textiles' action? •A. Time was not of the essence in the Mechanic-Textiles contract. •B. Mechanic had no reason to foresee on March 1 that Knitwear would suffer consequential damages in the amount of $25,000. •C. By entering into the Knitwear contract while knowing that its knitting machine was being repaired, Textiles assumed the risk of any delay loss to Knitwear. •D. In all probability, the liquidated damages paid by Textiles to Knitwear are not the same amount as the actual damages sustained by Knitwear in consequence of Textiles' late delivery of the cloth.

B

•On September 1, Team and Legs form a contract under which Legs will play basketball for Team during the months January to April of the following year, and Team will pay Legs a salary of $1 million monthly, on the thirtieth day of each such month. On September 15, Legs announces to Team that he will not, under any condition, play for Team, and that he will not report to work in January. Team takes no action in response. On January 1, Legs reports for play as required by his contract, announcing that he has changed his mind and that he will honor it after all. Team Owner advises Legs that he is not welcome to play, that he will not be paid a salary and that he is "fired." Which party has an action against which party? •A. Team has an action against Legs for total breach, arising from the anticipatory repudiation Legs made on September 15. •B. Legs has an action against Team arising from Team Owner's declaration, on January 1, that Legs was "fired." •C. Each party has an action against the other because each has manifested an intention not to honor the relevant contractual obligations. •D. Neither party has an action against the other because each has behaved improperly as to the other.

B

•Seller is a homeowner who has a boat for sale. Buyer 1 contracts to buy the boat for $12,587. Before the date for delivery, Buyer 1 breaches. Seller puts an ad in the paper and a week after the breach, Seller is able to sell the boat to Buyer 2 for $12,587. Of the following expenses incurred by Seller, which one is not included as incidental damages? •A) $20 for the ad in the paper. •B) $250 to consult an attorney about Buyer 1's breach. •C) $10 for insurance costs between the time of the breach and the resale. •D) $20 for storage costs between the time of the breach and the resale.

B

•Tenant rented a commercial building from Landlord, andoperated a business in it. The building's large front window was smashed by vandals six months before expiration of the Tenant-Landlord lease. Tenant, who was obligated thereunder to effect and pay for repairs in such cases, promptly contracted with Glazier to replace the window for $2,000, due 30 days after satisfactory completion of the work. Sixty days later, Tenant mailed a $1,000 check to Glazier bearing on its face the following conspicuous notation: "This check is in full and final satisfaction of your $2,000 window replacement bill." Without noticing this notation, Glazier cashed the check and now sues Tenant for the $1,000 difference. If Tenant's only defense is accord and satisfaction, is Tenant likely to prevail? •A. No, because Glazier failed to notice Tenant's notation on the check. •B. No, because the amount owed by Tenant to Glazier was liquidated and undisputed. •C. Yes, because by cashing the check Glazier impliedly agreed to accept the $1,000 as full payment of its claim. •D. Yes, because Glazier failed to write a reservation-of-rights notation on the check before cashing it.

B

•The owner of a store in a small beach town contacted a manufacturer about buying 100 Adirondack chairs. The parties reduced their agreement to sell and buy the 100 chairs to a signed writing that contained all of the essential terms, as well as a merger clause. When the chairs were delivered to the store owner, she discovered that they were not "Adirondack chairs," as she understood that term and as that term was commonly used, but a different style of chair altogether that was manufactured in the Adirondack region of New York. The store owner refused delivery, and the manufacturer sued the store owner for breach of contract, arguing that he complied with the terms of the contract. Will the store owner be allowed to introduce evidence of the meaning of the term "Adirondack chair"? •A. Yes, because the parol evidence rule does not apply to subsequent modifications. •B. Yes, because of the dispute between the parties over the meaning of the term "Adirondack chair." •C. No, because the written agreement of the parties was a complete integration. •D. No, because the contract was for a sale of goods between merchants.

B

•The proprietor of a food brokerage entered into oral negotiations with a manufacturer of gourmet food products for restaurants and select retail outlets. The proprietor wished to secure an exclusive distributorship for the manufacturer's products in the six New England states. At the end of the first stage of oral negotiations, the parties had come to an agreement on the major points, and only a few minor points of disagreement remained. Both, however, were anxious to begin distribution of the food products in New England, and the manufacturer assured the proprietor, "Don't worry about it; we'll work these things out." Assuming from this that he would be the New England distributor for the food products, the proprietor leased larger facilities, bought a number of trucks, and hired new workers. Shortly thereafter, the manufacturer informed the proprietor that another distributor, and not the proprietor, would receive the New England distributorship. If the proprietor prevails in a suit against the manufacturer, it will most likely be because the court applies which of the following theories? •A) Implied-in-fact contract. •B) Promissory estoppel. •C) Unjust enrichment. •D) Quasi-contract.

B

•WidgeCo, a manufacturer of widgets, sent an offer to Discorp, a major wholesaler. WidgeCo offered a standard lot (quantity well-known in the widget trade) of widgets for $8,000. This was a good price, so the president of Discorp personally mailed back to WidgeCo Discorp's standard printed acceptance form. However, the president wrote in large letters in his own hand on the form, "Our liability on this contract is limited to $200." Two days later, the WidgeCo sales manager received the communication from Discorp. WidgeCo sent no additional communication to Discorp. Assuming no additional facts, what is the relationship between the parties? •A. There is no contract between WidgeCo and Discorp because Discorp made a material alteration. •B. There is a valid, enforceable contract between WidgeCo and Discorp, but it is limited to the terms of WidgeCo's offer. •C. There is a valid, enforceable contract between WidgeCo and Discorp and it contains the additional term because WidgeCo failed to object. •D. Discorp has sent a valid counteroffer to WidgeCo which WidgeCo can accept or reject.

B

•1. On July 15, in a writing signed by both parties, Fixtures, Inc., agreed to deliver to Druggist on August 15 five storage cabinets from inventory for a total price of $5,000 to be paid on delivery. On August 1, the two parties orally agreed to postpone the delivery date to August 20. On August 20, Fixtures tendered the cabinets to Druggist, who refused to accept or pay for them on the ground that they were not tendered on August 15, even though they otherwise met the contract specifications. Assuming that all appropriate defenses are seasonably raised, will Fixtures succeed in an action against Druggist for breach of contract? •A. Yes, because neither the July 15 agreement nor the August 1 agreement was required to be in writing. •B. Yes, because the August 1 agreement operated as a waiver of the August 15 delivery term. •C. No, because there was no consideration to support the August 1 agreement. •D. No, because the parol evidence rule will prevent proof of the August 1 agreement.

B.

Seller breaches, Buyer keeps the goods

Breach occurs when: buyers accepts defective goods Damages: difference between value of goods as accepted and value as promised as promised plus incidental damages and consequential damages Fair market value if perfect minus fair market value as delivered

UCC Damages

Buyer's remedies 1. seller breaches-seller has goods 2. seller breaches- buyer has goods Seller's remedies 1. buyer breaches-buyer has goods 2. buyer breaches-seller has goods

A 17 year old boy walked into a medical clinic and requested assistance with a deep cut he received when he fell off his bike. •The doctor told the boy that if he agreed to work at the clinic for 45 hours a week for four weeks, he would stitch the wound and apply a medicated bandage. The clinic typically charges $225 for such treatment. Although it seemed like a lot of work for $225, the boy needed immediate medical attention, so he accepted the offer and promised to report for work the next day, after which the doctor treated the boy's injury. On his way home from the clinic, the medicated bandage fell into a ditch and was lost for good. The boy refused to report for work the next day. If the medical clinic sues the boy for $225 and loses, it will be because: A. The boy was a minor. •B. The medicated bandage was destroyed and thus there was a breach of the implied warranty of merchantability. •C. The contract was unconscionable. •D. It was impossible for the boy to perform.

C

•. A commercial buyer receives a shipment of goods from a seller and stores them without inspecting them. Three months later, the buyer discovers a defect in the goods. The buyer immediately informs the seller that it is rejecting the goods and sends them back. Is the rejection a rightful rejection? •A. Yes, because the buyer had a reasonable time to conduct an inspection. •B. Yes, because the buyer could revoke its acceptance. •C. No, because the buyer had accepted the goods. •D. No, because the buyer has waived its right to recover damages from the seller.

C

•. A farmer orders a water pump that the seller promises will pump 1,000 gallons per minute. The farmer inspects the pump on arrival and finds that it will pump only 950 gallons per minute. The seller promises to make it work properly, so the farmer accepts it and pays for it. Later, it becomes clear that the seller cannot improve the pumping capacity but had hoped that the farmer would accept the slightly reduced capacity. In fact, unknown to the seller, the farmer requires the full capacity to fully irrigate his fields. Can the farmer revoke his acceptance? •A. Yes, because the nonconformity substantially impairs its value to him. •B. Yes, because he accepted it on the reasonable assumption that its nonconformity would be cured and it has not been seasonably cured. •C. Yes, because both elements described in responses A and B are satisfied. •D. No, because once there is acceptance it is too late to reject.

C

•. Carl goes to a store to buy a television. At the store, there's a television playing. Carl thinks the picture and sound on that television are good, so he buys the same model. He takes it home, plugs it in, and finds that the picture and sound, while reasonably good, aren't as good as what he saw in the store. He sues for breach of warranty. If he wins, it will be because •A. there's a failure of consideration. •B. there's a breach of the warranty of fitness for a particular purpose. •C. the demonstration of the model constituted a warranty that proved false. •D. there's a breach of the implied warranty of merchantability.

C

•. During negotiations, the seller of a boat promised the buyer that the engine was warranted to be free of defects for a period of two years. The parties eventually signed a written agreement that contained no express warranty terms. Fourteen months after the purchase, the engine developed problems because of a defect. A court found that the writing is a complete and exclusive statement of the terms of agreement. Is the seller liable under the oral warranty? •A) It remains to be determined as a question of fact whether the promise was made. •B) Yes, because the oral promise does not contradict the terms of the writing. •C) No, because the writing was a complete and exclusive expression of their agreement. •D) No, because the buyer would have a claim even under the written warranty.

C

•. During negotiations, the seller of a boat promises the buyer that the engine was warranted to be free of defects for a period of two years. The parties then sign a written agreement that contains a warranty term with standard disclaimers of express and implied warranties, followed by language stating that "seller warrants that the engine will be free of defects for a period of one year." After 14 months, the engine develops problems because of a defect. A court finds that the writing is a partial integrationand that the promise was actually made. Is the seller liable under the warranty? •A) Yes, because the parties did not intend their writing to be final as to the warranty. •B) Yes, because the oral promise does not contradict the terms of the writing. •C) No, because the writing was a final expression of their agreement as to the warranty. •D) No, because the buyer would have a claim even under the written warranty.

C

•. T, the manager of a state fair, contracted with S, a renowned hog breeder, to exhibit S's world champion animal, Megahawg, for three weeks of the annual fair, at the conclusion of which S would receive an honorarium of $300. Two days before the opening of the fair, Megahawg took sick with boarsitus, a communicable disease among swine and, under applicable state quarantine laws, very probably could not be exhibited for at least a month. Upon learning this, T can legally pursue which of the following courses of action with respect to his contract with S? •A. Suspend his own performance, demand assurances from S, and treat a failure by S to give them as an actionable repudiation. •B. Suspend his own performance and recover damages from S for breach of contract unless S at once supplies an undiseased hog as a substitute for Megahawg. •C. Terminate his own performance and treat Megahawg's illness as discharging all remaining duties under the contract. •D. Terminate the contract, but only if he (T) seeks promptly to obtain for the exhibit a suitable substitute for Megahawg from another hog owner.

C

•3. On March 31, Selco and Byco entered into a written agreement in which Selco agreed to fabricate and sell to Byco 10,000 specially designed brake linings for a new type of power brake manufactured by Byco. The contract provided that Byco would pay half of the purchase price on May 15 in order to give Selco funds to "tool up" for the work; that Selco would deliver 5,000 brake linings on May 31; that Byco would pay the balance of the purchase price on June 15; and that Selco would deliver the balance of the brake linings on June 30. On May 10, Selco notified Byco that it doubted that it could perform because of problems encountered in modifying its production machines to produce the brake linings. On May 15, however, Selco assured Byco that the production difficulties had been overcome, and Byco paid Selco the first 50 percent installment of the purchase price. Selco did not deliver the first 5,000 brake linings on May 31, or at any time thereafter; on June 10, Selco notified Byco that it would not perform the contract. Which of the following correctly states Byco's rights and obligations immediately after receipt of Selco's notice on May 10? •A. Byco could treat the notice as an anticipatory repudiation, and had a cause of action on May 10 for breach of the entire contract. •B. Byco could treat the notice as an anticipatory repudiation, and could sue to enjoin an actual breach by Selco on May 31. •C. Byco had no cause of action for breach of contract, but could suspend its performance and demand assurances that Selco would perform. •D. Byco had no cause of action for breach of contract, and was required to pay the installment of the purchase price due on May 15 to preserve its rights under the contract.

C

•A builder contracted with a landowner to construct a house on the landowner's property for a contract price of $100,000. The landowner committed a total breach of the contract at a time when the builder had already incurred costs of part performance of $30,000 and would have to spend an additional $60,000 to finish the job. The builder is entitled to recovery in the amount of: •A) $10,000. •B) $30,000. •C) $40,000. •D) $60,000.

C

•A farmer ordered a complex irrigation pump system from a seller. There was no disclaimer of the implied warranty of merchantability. When the pump arrived, the farmer installed it and found that it did not work properly. The farmer continued to try to get the pump to work and notified the seller of his efforts. Finally, after the farmer had the pump for two weeks, he returned it to the seller. The seller sued the farmer, claiming the attempted rejection was wrongful. Did the farmer wrongfully reject the pump? •A. Yes, because he had accepted it by keeping it for two weeks. •B. Yes, because the goods did not fail to conform to the contract. •C. No, because he had a reasonable period of time to inspect the goods. •D. No, because there was a material breach by the seller.

C

•A general contractor who wished to bid on a construction project solicited bids from a variety of subcontractors. Four electrical subcontractors submitted bids to the contractor in the amounts of $75,000, $85,000, $90,000, and $95,000, respectively. As he was making out his company's bid, which was higher than he wanted it to be, the contractor called the low bidder on the electrical work and told him, "We won't be able to do it with your present bid, but if you can shave off $5,000, I'm sure that the numbers will be there for us to get that project." The low bidder told the contractor that he could not lower his bid, adding that the bid he submitted was based on a $15,000 error, and he could not do the job for less than $90,000. The contractor lost the construction job and subsequently sued the low bidder. The low bidder is liable for: •A. Breach of contract, because the mistake was not so unreasonably obvious as to make acceptance of his bid unconscionable. •B. Breach of contract, because the mistake was unilateral. •C. Nothing, because the low bidder rejected the contractor's counteroffer. •D. Nothing, because even though the low bidder lacked authority to renege on its bid, the contractor suffered no damages because no bidder was willing to do the work for $70,000.

C

•A sporting goods shop contracted with a publisher to buy, for sale in its store, 1,200 posters featuring a professional golfer. During production, the image of the golfer was inadvertently reversed and the right-handed golfer appeared to be left-handed. When the posters were delivered on the date provided in the contract, the sporting goods shop noticed the discrepancy, which had no provable significant impact on the effectiveness of the poster. In the opinion of the shop management, however, the posters did not look as good as they had in the catalog from which the shop had ordered them. Is the sporting goods shop legally entitled to reject the posters? •(A) No, because the nonconformity does not materially alter the value of the posters to the sporting goods shop. •(B) No, because the publisher must be given an opportunity to cure the nonconformity before the sporting goods shop can reject the posters. •(C) Yes, because the posters do not conform to the contract. •(D) Yes, because the publisher has breached an implied warranty of fitness for a particular purpose.

C

•A wholesaler persuaded a retailer to order a line of dolls for the Christmas season, even though the retailer was skeptical of the dolls' marketability. The contract stated that the retailer would "pay $1,500 for its order of 100 dolls provided that the dolls sold during the Christmas season." Some dolls did sell, but on February 12, the retailer had 80 of them in inventory. He sent the wholesaler notice that he would be returning the 80 dolls. The wholesaler replied that it did not want the dolls back, and that the retailer should continue to try to sell them. Despite this reply, the retailer sent the wholesaler a check for $300 and shipped the dolls to the wholesaler, who refused to accept them but did accept the check. Thereafter, the retailer held the dolls at his warehouse. The wholesaler brought an action to recover the $1,200 balance. The wholesaler will: •A. Recover, because the retailer still has the dolls in his possession. •B. Recover, because it was not a condition precedent that the dolls be sold during the Christmas season, but merely a convenient time for payment. •C. Not recover, because sale during the Christmas season was a condition precedent to payment. •D. Not recover, because accepting the $300 waived any rights that the wholesaler may have had to enforce the contract.

C

•An employer offered to pay a terminated employee $50,000 to release all claims the employee might have against the employer. The employee orally accepted the offer. The employer then prepared an unsigned release agreement and sent it to the employee for him to sign. The employee carefully prepared, signed, and sent to the employer a substitute release agreement that was identical to the original except that it excluded from the release any age discrimination claims. The employer signed the substitute release without reading it. Shortly thereafter, the employee notified the employer that he intended to sue the employer for age discrimination. Is the employer likely to prevail in an action seeking reformation of the release to conform to the parties' oral agreement? •A. No, because the employer acted unreasonably by failing to read the substitute release prior to signing it. •B. No, because the parol evidence rule will preclude evidence of the oral agreement. •C. Yes, because the employee's fraudulent behavior induced the employer's unilateral mistake. •D. Yes, because the parties were mutually mistaken regarding the contents of the signed release.

C

•During an ice storm, a man's car slipped down an embankment and became lodged against a large tree. The man called a towing company and told the company's manager that the car was 100 feet down the embankment. "That's lucky," said the manager, "because our winch only goes 100 feet." After the manager and the man agreed on a price, an employee of the company attempted to reach the car but could not because the car turned out to be 120 feet down the embankment. Is the towing company's performance excused on the grounds of mistake? •A. No, because both parties were uncertain about the distance. •B. No, because the towing company assumed the risk by the manager's failure to examine the distance himself. •C. Yes, because at the time of contracting, both parties were mistaken about a basic assumption on which the contract was based. •D. Yes, because the agreement did not allocate the risk of mistake to either party.

C

•In a single writing, Painter contracted with Farmer to paint three identical barns on her rural estate for $2,000 each. The contract expressly provided for Farmer's payment of $6,000 upon Painter's completion of the work on all three barns. Painter did not ask for any payment when the first barn was completely painted, but she demanded $4,000 after painting the second barn. Assume that Farmer rightfully refused Painter's demand for payment. If Painter immediately terminates the contract without painting the third barn, what is Painter entitled to recover from Farmer? •A. Nothing, because payment was expressly conditioned on completion of all three barns. •B. Painter's expenditures plus anticipated "profit" in painting the first two barns, up to a maximum recovery of $4,000. •C. The reasonable value of Painter's services in painting the two barns, less Farmer's damages, if any, for Painter's failure to paint the third barn. •D. The amount that the combined value of the two painted barns has been increased by Painter's work. • •Which answer would be correct if the italicized sentence was omitted from the contract? -(B)

C

•In early July, the owner of a toy store entered into a contract with a craftsman for the purchase of 25 hand-carved wooden rocking horses for the holiday sales season. The agreement failed to state a delivery date for the horses. After several months went by without any word from the craftsman, the owner forgot about the agreement. On October 15, the owner ordered a quantity of mass-produced resin-based rocking horses to sell in his store for the upcoming holiday season. One week later, the craftsman arrived at the toy store with a truck carrying the horses and demanded payment, but the toy store owner refused, stating that he already had more than enough rocking horses to sell. The craftsman sued the owner for breach of contract. May the craftsman enforce the contract? •A. No, because the contract was missing an essential term, the delivery date, at the time of formation. •B. No, because so much time had passed without word from the craftsman that the toy store owner had reasonable grounds for insecurity and was entitled to cancel the contract and mitigate his damages. •C. Yes, because the delivery date was reasonable given the nature of the goods. •D. Yes, because the craftsman accepted the contract by completing the hand-carved horses.

C

•Jenny, a general contractor, advertised in a trade publication that she planned to bid on the construction of a new building to be located in the Civic Mall. The advertisement welcomed bids from subcontractors to perform various functions, such as plumbing, electrical work, and masonry. The lowest plumbing bid was from Plunger, who bid $10,000. Jenny used Plunger's bid in preparing her general bid. At 2 p.m. on June 22, Jenny submitted her general bid. At 3 p.m. Plunger called her and said, "I'm sorry, Jenny, but I made a mistake on that bid I submitted to you; I can't possibly do that plumbing work for a dime less than $12,000." Jenny told him, "Look, you've done a lot of good work for me in the past and we all make mistakes. I'll just forget you ever made that $10,000 bid." Plunger effusively thanked Jenny. Jenny then hired Flusher to do the plumbing work for $12,000. She then sued Plunger for damages. Jenny will: •A. Win, because there was no additional consideration to support a release. •B. Win, because the Statute of Frauds applies to the agreement. •C. Lose, because a mutual rescission has taken place. •D. Lose, because Plunger detrimentally relied on the release.

C

•Kontractor agreed to build a power plant for a public utility. Subbo agreed with Kontractor to lay the foundation for $200,000. Subbo supplied goods and services worth $150,000, for which Kontractor made progress payments aggregating $100,000 as required by the subcontract. Subbothen breached by refusing unjustifiably to perform further. Kontractor reasonably spent $120,000 to have the work completed by another subcontractor. Subbo sues Kontractorfor the reasonable value of benefits conferred, and Kontractorcounterclaims for breach of contract. Which of the following should be the court's decision? •A. Subbo recovers $50,000, the benefit conferred on Kontractorfor which Subbo has not been paid. •B. Subbo recovers $30,000, the benefit Subbo conferred on Kontractor minus the $20,000 in damages incurred by Kontractor. •C. Kontractor recovers $20,000, the excess over the contract price that was paid by Kontractor for the performance it had bargained to receive from Subbo. •D. Neither party recovers anything, because Subbo committed a material, unexcused breach and Kontractor received a $50,000 benefit from Subbo for which Subbo has not been paid.

C

•Same facts as the prior slide, but for this question only, assume the following facts. Buyer unequivocally repudiated the contract on August 1. On August 15, Seller urged Buyer to change her mind and proceed with the scheduled closing on December 1. On October 1, having heard nothing further from Buyer, Seller sold and conveyed his ranch to Rancher without notice to Buyer. On December 1, Buyer attempted to close under the June 1 contract by tendering the full purchase price to Seller. Seller rejected the tender. If Buyer sues Seller for breach of contract, Buyer will probably •A. win, because Seller failed seasonably to notify Buyer of any pending sale to Rancher. •B. win, because Seller waived Buyer's August 1 repudiation by urging her to retract it on August 15. •C. lose, because Buyer did not retract her repudiation before Seller materially changed his position in reliance thereon by selling the ranch to Rancher. •D. lose, because acceptance of the purchase price by Seller was a concurrent condition to Seller's obligation to convey the ranch to Buyer on December 1.

C

•Which of the following requirements must be met for modification of a sales contract under the Uniform Commercial Code? •(A) There must be consideration present if the contract is between merchants. •(B) There must be a writing if the original sales contract is in writing. •(C) The modification must satisfy the Statute of Frauds if the contract as modified is within its provisions. •(D) The parol evidence rule applies and thus a writing is required.

C

•Zeller contracted in writing to deliver to Baker 100 bushels of wheat on August 1 at $3.50 per bushel. Because his suppliers had not delivered enough wheat to him by that time, Zeller on August 1 had only 95 bushels of wheat with which to fulfill his contract with Baker. If Zeller tenders 95 bushels of wheat to Baker on August 1, and Baker refuses to accept or pay for any of the wheat, which of the following best states the legal relationship between Zeller and Baker? •A. Zeller has a cause of action against Baker because Zeller has substantially performed his contract. •B. Zeller is excused from performing his contract because of impossibility of performance. •C. Baker has a cause of action against Zeller for Zeller's failure to deliver 100 bushels of wheat. •D. Baker is obligated to give Zeller a reasonable time to attempt to obtain the other five bushels of wheat.

C

•seller agrees to sell a television set to a buyer for $1,000. After the deal was completed, the buyer discovers that other sellers in the area were selling the same television set for $750. The buyer claims that he is entitled to rescind the contract because the price must be reasonable. Is the buyer correct? •A) Yes, because the purpose of the UCC is to ensure that prices are reasonable. •B) Yes, because the market price is the reasonable price and this price is above market. •C) No, because the reasonable price rule only applies when nothing is said as to price. •D) No, because freedom of contract always allows a buyer to agree to pay an unreasonable price.

C

•. Alpha and Beta entered into a written agreement in which Alpha agreed to sell and Beta agreed to buy a tract of land. After they signed the agreement but before closing, they got into an argument. Alpha decided the price was too low and Beta decided the price was too high. As a result, they both decided to cancel the contract. Later Beta changed his mind and decided he would pay Alpha the contract price for the property. He contacted Alpha who refused to sell to him at any price. If Beta brings suit against Alpha for specific performance, Alpha's best defense is that •(A) Beta did not give consideration for the original agreement. •(B) Beta did not retract his waiver before any detrimental reliance by Alpha. •(C) The contract was discharged by intervening illegality. •(D) The contract was discharged by mutual rescission.

D

•5. A buyer orders 100 widgets for $10,000. When the widgets arrive, he discovers that one of them is broken in the box. He has found another source that will sell him 100 widgets for $9,600, so he figures he can reject the widgets, buy them from the other source, and still come out ahead. He asks you for advice. Can he rightfully reject the widgets? •A. Yes, because the goods fail to conform to the contract. •B. Yes, because the seller has materially breached. •C. No, because the seller did not breach. •D. No, because his rejection would not be in good faith. • •UCC 1-304. Every contract or duty within the UCC imposes an obligation of good faith in its performance and enforcement. UCC 1-201(20) "Good faith"...means honesty in factandthe observance of reasonable commercial standards of fair dealing

D

•. Swatter, a baseball star, contracted with the Municipal Symphony Orchestra, Inc., to perform for $5,000 at a children's concert as narrator of "Peter and the Wolf." Shortly before the concert, Swatter became embroiled in a highly publicized controversy over whether he had cursed and assaulted a baseball fan. The orchestra canceled the contract out of concern that attendance might be adversely affected by Swatter's appearance. Swatter sued the orchestra for breach of contract. His business agent testified without contradiction that the cancellation had resulted in Swatter's not getting other contracts for performances and endorsements. The trial court instructed the jury, in part, as follows: "If you find for the plaintiff, you may award damages for losses which at the time of contracting could reasonably have been foreseen by the defendant as a probable result of its breach. However, the law does not permit recovery for the loss of prospective profits of a new business caused by breach of contract." On Swatter's appeal from a jury verdict for Swatter, and judgment thereon, awarding damages only for the $5,000 fee promised by the orchestra, the judgment will probably be •A. affirmed, because the trial court stated the law correctly. •B. affirmed, because the issue of damages for breach of contract was solely a jury question. •C. reversed, because the test for limiting damages is what the breaching party could reasonably have foreseen at the time of the breach. •D. reversed, because under the prevailing modern view, lost profits of a new business are recoverable if they are established with reasonable certainty.

D

•. The Kernel Corporation, through its president, Gritz, requested from Vault Finance, Inc., a short-term loan of $100,000. On April 1, Gritz and Vault's loan officer agreed orally that Vault would make the loan on the following terms: (1) The loan would be repaid in full on or before the following July 1 and would carry interest at an annual rate of 5%; and (2) Gritz would personally guarantee repayment. The loan was approved and made on April 5. The only document evidencing the loan was a memorandum, written and supplied by Vault and signed by Gritz for Kernel, that read in its entirety: -"April 5 -In consideration of a loan advanced on this date, Kernel Corporation hereby promises to pay Vault Finance, Inc., $100,000 on September 1. -Kernel Corporation -By /s/ Demeter Gritz -Demeter Gritz, President" •Kernel Corporation did not repay the loan on or before July 1, although it had sufficient funds to do so. On July 10, Vault sued Kernel as principal debtor and Gritz individually as guarantor for $100,000, plus 5% interest from April 5. At the trial, can Vault prove Kernel's oral commitment to repay the loan on or before July 1? •(A) Yes, because the oral agreement was supported by an independent consideration. •(B) Yes, because the evidence of the parties' negotiations is relevant to their contractual intent concerning maturity of the debt. •(C) No, because such evidence is barred by the preexisting duty rule. •(D) No, because such evidence contradicts the writing and is barred by the parol evidence rule.

D

•. The owner of a house put an ad for its sale in the paper. Her neighbor saw the ad and told her that he wanted to buy the house but had to arrange for financing. The owner suggested that they write a contract for sale then and there so that they would not have to waste any time while he got his financing. They orally agreed that the contract would not become binding unless the neighbor obtained financing, but the written contract did not mention this and appeared to be a fully integrated document. The neighbor could not obtain financing and the owner brings suit to enforce the written contract. Who will prevail? •A. The owner, because the contract was a fully integrated writing. •B. The owner, because parol evidence is not allowed to contradict a writing. •C. The neighbor, because the oral agreement that the contract would not be binding if the neighbor did not get financing was made contemporaneous with the writing. •D. The neighbor, because obtaining financing was a condition precedent.

D

•. Tune Corporation, a radio manufacturer, and Bill's Comex, Inc., a retailer, after extensive negotiations entered into a final, written agreement in which Tune agreed to sell and Bill's agreed to buy all of its requirements of radios, estimated at 20 units per month, during the period January 1, 1988, and December 31, 1990, at a price of $50 per unit. A dispute arose in late December, 1990, when Bill's returned 25 undefective radios to Tune for full credit after Tune had refused to extend the contract for a second three-year period. In an action by Tune against Bill's for damages due to return of the 25 radios, Tune introduces the written agreement, which expressly permitted the buyer to return defective radios for credit but was silent as to return of undefective radios for credit. Bill's seeks to introduce evidence that during the three years of the agreement it had returned, for various reasons, 125 undefective radios, for which Tune had granted full credit. Tune objects to the admissibility of this evidence. The trial court will probably rule that the evidence proffered by Bill's is •(A) inadmissible, because the evidence is barred by the parol evidence rule. •(B) inadmissible, because the express terms of the agreement control when those terms are inconsistent with the course of performance. •(C) admissible, because the evidence supports an agreement that is not within the relevant statute of frauds. •(D) admissible, because course-of-performance evidence, when available, is considered the best indication of what the parties intended the writing to mean.

D

•11. Assume the same facts as the prior slide. Which of the following, if proved, would best support Mural's defense? •(A) Gennybelle gave Mural no consideration for an irrevocable sub-bid. •(B) Mural's sub-bid expressly requested Gennybelle's acceptance after awarding of the main contract. •(C) Even after paying $18,000 for the paperhanging, Gennybelle would make a net profit of $100,000 on the Doctors' Building contract. (D) Before submitting her own bid, Gennybelle had reason to suspect that Mural had made a computational mistake in figuring his sub-bid

D

•4. Same facts as the prior slide. Which of the following is NOT a correct statement of the parties' legal status immediately after Selco's notice on June 10? •A. Byco has a cause of action for total breach of contract because of Selco's repudiation, but that cause of action will be lost if Selco retracts its repudiation before Byco changes its position or manifests to Selco that Byco considers the repudiation final. •B. Byco can bring suit to rescind the contract even if it elects to await Selco's performance for a commercially reasonable time. •C. Byco can await performance by Selco for a commercially reasonable time, but if Byco awaits performance beyond that period, it cannot recover any resulting damages that it reasonably could have avoided. •D. Byco has a cause of action for breach of contract that it can successfully assert only after it has given Selco a commercially reasonable time to perform.

D

•7. Neighbors of an apparently destitute couple bought a month's supply of food and gave it to them. Later, the wife confided in the neighbors that she and her husband did have money and that, because they had been so kind, she was leaving them money in her will. When the wife died, at the neighbors' request the husband gave the neighbors the following signed instrument: "In consideration of my wife's promise to our neighbors, and their agreement not to sue her estate, I agree to pay them the sum of $5,000." When the husband died of a heart attack several days later, the neighbors asked the administrator of his estate to pay them the $5,000. The administrator refused on the ground that there was no consideration for the agreement. On which of the following theories would it be most likely that the neighbors would recover? • A. The husband's written instrument was a binding unilateral contract. • B. The husband's acceptance of the food was fraudulent. • C. The husband is bound by promissory estoppel. • D. The husband and the neighbors entered into a valid compromise.

D

•A homeowner contracted with a local builder, a sole proprietorship, for the builder to personally build a wooden deck onto the back of her house. The contract called for half of the contract price of $20,000 to be paid to the contractor before he began work and the other half to be paid to him when the job was completed. The contractor began the work but, partway through the job, he got an offer for a rush job that paid better and abruptly quit. The homeowner sues the contractor for specific performance. Will she prevail? •A. Yes, because there has been a novation. •B. Yes, because the contract between the parties was valid and the contractor had no legal justification for abruptly quitting. •C. No, because by not paying the contractor for the second half of the job, the homeowner has not satisfied all of her conditions under the contract. •D. No, because the contract is for personal services.

D

•A manufacturer of electronic game cartridges for use with other manufacturers' electronic game systems received a letter from a buyer for a large toy store chain ordering two 100-unit lots of a particular game cartridge from the manufacturer's catalog. The buyer's letter stated that the cartridges must be compatible with two specified game systems. The manufacturer made no warranties of any type regarding the compatibility of the cartridges with any game systems, but both parties reasonably believed that the cartridges were compatible with both game systems. On receiving the shipment, the buyer discovered that the cartridges did not work with one of the game systems. The buyer has made full payment to the manufacturer. Which of the following is the most appropriate remedy for the buyer? •A. Specific performance of the contract, requiring the manufacturer to ship two 100-unit lots of cartridges compatible with both game systems. •B. Rescission of the contract. •C. Restitution by the manufacturer of the purchase price of the two lots. D.Both B and C.

D

•A popular singer expressed interest in performing in a theater producer's musical production. The producer and the singer agreed that there would be no enforceable contractual obligations between them until they had entered into a final, signed contract. They then began negotiating the terms of the contract. While the parties were negotiating, the producer began to spend money on the production. When the parties were unable to agree on the terms of a final contract, the producer made reasonable but unsuccessful efforts to hire another singer before abandoning the production. If the producer sues the singer, what will he likely recover? •A. Expectation damages in the form of the profits that the production would have made if it had been performed. •B. Reliance damages in the form of the actual expenses the producer incurred while negotiating with the singer. •C. Restitution for any harm done to the producer's reputation when the production was abandoned. •D. Nothing, because it was not reasonable for the producer to expect to be reimbursed for expenses incurred when he knew that the singer was not obligated until there was a final, signed agreement.

D

•A seller of supercomputers represents that a certain model computer can perform certain specified functions. A buyer buys that model at a price of $25,000. It turns out that although the model is a very good computer, it cannot perform the specified functions. The computer as delivered to the buyer is worth $20,000. If it did have the specified functions, it would be worth $75,000. If the buyer keeps the computer and claims damages, how much can it recover? A. $0 B. $5,000 C. $50,000 D. $55,000

D

•A woman went to her local department store and told the salesperson that she wanted a coat that was extremely warm. The salesperson went into his stockroom and brought out four different styles of very warm coats. The woman tried on each of the four but did not like the way any of them looked. While walking around the store, however, the young woman saw a coat she did like and told the salesperson to bring one in her size. The salesperson brought her the coat and he said that it was made of the finest cashmere and would probably last for years. The young woman tried on the coat and told the salesperson that she would take it and paid him. After wearing the coat twice, however, she decided it was not warm enough for her climate. She took the coat back to the department store and demanded her money back. The store refused. If the woman sues to get her money back, under which theory would she most likely prevail? • A. Breach of the implied warranty of fitness for particular purpose. • B. Breach of the implied warranty of merchantability. • C. Breach of express warranty. • D. None of the listed warranties.

D

•Albert engaged Bertha, an inexperienced actress, to do a small role in a new Broadway play for a period of six months at a salary of $800 a week. Bertha turned down another role in order to accept this engagement. On the third day of the run, Bertha was hospitalized with influenza and Helen was hired to do the part. A week later, Bertha recovered, but Albert refused to accept her services for the remainder of the contract period. Bertha then brought an action against Albert for breach of contract. Which of the following is Bertha's best legal theory? •(A) Her acting contract with Albert was legally severable into weekly units. •(B) Her performance of the literal terms of the contract was physically impossible. •(C) Her reliance on the engagement with Albert by declining another acting role created an estoppel against Albert. •(D) Her failure to perform for one week was not a material failure so as to discharge Albert's duty to perform.

D

•Computers, Inc. contracted in writing with Bank to sell and deliver to Bank a mainframe computer using a new type of magnetic memory, then under development but not perfected by Computers, at a price substantially lower than that of a similar computer using current technology. The contract's delivery term was "F.O.B. Bank, on or before July 31." After making the contract with Bank, Computers discovered that the new technology it intended to use was unreliable and that no computer manufacturer could yet build a reliable computer using that technology. Computers thereupon notified Bank that it was impossible for Computers or anyone else to build the contracted-for computer "in the present state of the art." If Bank sues Computers for failure to perform its computer contract, the court will probably decide the case in favor of •A. Computers, because its performance of the contract was objectively impossible. •B. Computers, because a contract to build a machine using technology under development imposes only a duty on the builder to use its best efforts to achieve the result contracted for. •C. Bank, because the law of impossibility does not apply to merchants under the applicable law. •D. Bank, because Computers assumed the risk, in the given circumstances, that the projected new technology would not work reliably.

D

•John, a law student, is selling his car to Mary. He says, "This car will get 25 miles per gallon around town. Of course, I disclaim all express warranties." The parties then enter into a partially integrated agreement that makes no mention of warranties or warranty disclaimers. Has John given Mary a warranty that the car will get 25 miles per gallon around town? •A. No, because John is not a merchant seller. •B. No, because the statement is puffing. •C. No, because all express warranties are disclaimed. D.Yes.

D

•On April 1, Owner offered Buyer the right to purchase Greenacre for $100,000; Owner promised to hold the offer open for 30 days. On April 20, Owner, having received no communication from Buyer, sold and conveyed Greenacre to Citizen for $120,000. On April 21, Owner received a letter from Buyer stating, "I am hereby accepting your offer to sell Greenacre and am prepared to close whenever you're ready." Assume that Buyer prevails in a suit against Owner. Which of the following is Buyer entitled to recover? •A. Nominal damages only, because the remedy of specific performance was not available to Buyer. •B. The fair market value, if any, of an option to purchase Greenacre for $100,000. •C. $100,000, which represents the contract price. •D. The amount, if any, by which the fair market value of Greenacre on the date of Owner's breach exceeded $100,000.

D

•On June 1, Seller and Buyer contracted in writing for the sale and purchase of Seller's cattle ranch (a large single tract), and to close the transaction on December 1. For this question only, assume the following facts. On October 1, Buyer told Seller, "I'm increasingly unhappy about our June 1 contract because of the current cattle market, and do not intend to buy your ranch unless I'm legally obligated to do so." If Seller sues Buyer on October 15 for breach of contract, Seller will probably •A. win, because Buyer committed a total breach by anticipatory repudiation on October 1. •B. win, because Buyer's October 1 statement created reasonable grounds for Seller's insecurity with respect to Buyer's performance. •C. lose, because the parties contracted for the sale and conveyance of a single tract, and Seller cannot bring suit for breach of such a contract prior to the agreed closing date. •D. lose, because Buyer's October 1 statement to Seller was neither a repudiation nor a present breach of the June 1 contract.

D

•On September 1, a law school professor announced to his class that he would pay the tuition of the bar review course of the student's choice for the student who received the highest grade in his Constitutional Law class. A week later, one student in the professor's class told the professor that he was giving his best effort to get the highest grade in class to win the prize and had already purchased every substantive Constitutional Law outline on the market. On October 1, the professor told his class that he was withdrawing his offer. Which of the following statements about the professor's October 1 announcement is correct? •A. It had no legal effect, because no offer had been made. •B. It was an effective revocation of the offer to all students who heard it because it was made in the same manner as the offer. •C. It was an ineffective revocation as to any student who failed to hear it. •D. It was ineffective as to the student who had begun performance prior to the withdrawal of the offer.

D

•Owner's dog has run away from home. On June 1, Finder finds Owner's dog. On June 2, Owner posts a reward in the local newspaper. On June 3, Finder returns the dog to Owner but does not receive a reward. On June 4, Finder learns (for the first time) of the reward and telephones Owner stating, "I accept." Do the parties have a contract and, if so, when was it formed? •A. The parties have a contract and it was formed on June 1. •B. The parties have a contract and it was formed on June 3. •C. The parties have a contract and it was formed on June 4. •D. The parties do not have a contract.

D

•Pursuant to a contract between them, a seller ships a widget to the buyer. The contract price of the widget is $10,000. After inspection, the buyer determines that the widget appears to be defective. The buyer incurs expenses of $500 in hiring an expert to inspect the widget, and after receiving the expert's report that the widget is damaged beyond repair, the buyer incurs an expense of $300 shipping it back to the seller. The buyer then pays a commission of $200 to a broker to find a replacement widget, and buys one at the cost of $11,000. Which of the expenses can the buyer not recover as incidental damages? •A) The $500 expense to have an expert inspect the widget. •B) The $300 expense of shipping the widget back to the seller. •C) The $200 expense of paying a broker to find a replacement widget. •D) The $1,000 difference between the price of the original widget and the replacement widget.

D

•Same facts as the prior slide, but for this question only make the following assumptions. On March 1, Fixtures tendered 24 sets to Apartments and explained, "One of the 25 sets was damaged in transit from the manufacturer to us, but we will deliver a replacement within 5 days." Which of the following statements is correct? •A. Apartments is entitled to accept any number of the 24 sets, reject the rest, and cancel the contract both as to any rejected sets and the lot due on April 1. •B. Apartments is entitled to accept any number of the 24 sets and to reject the rest, but is not entitled to cancel the contract as to any rejected sets or the lot due on April 1. •C. Apartments must accept the 24 sets but is entitled to cancel the rest of the contract. D.Apartments must accept the 24 sets and is not entitled to cancel the rest of the contract.

D

•The owner of a parcel of land received the following letter from a buyer: "I will pay you $2,200 an acre for [the parcel]." The owner's letter of reply stated, "I accept your offer." Unknown to the owner, the buyer had intended to offer only $2,000 per acre but had mistakenly typed "$2,200." As both parties knew, comparable land in the vicinity had been selling at prices between $2,000 and $2,400 per acre. Which of the following states the probable legal consequences of the correspondence between the parties? •(A) There is no contract, because the parties attached materially different meanings to the price term. •(B) There is no enforceable contract, because the buyer is entitled to rescission due to a mutual mistake as to a basic assumption. •(C) There is a contract formed at a price of $2,000 per acre, as the buyer intended. •(D) There is a contract formed at a price of $2,200 per acre, regardless of the buyer's true intention.

D

•The owner of an apartment building contracted with a painter to paint the porches of the apartments for $5,000. The contract was specifically made subject to the owner's good faith approval of the work. The painter finished painting the porches. The owner inspected the porches and believed in good faith that the painter had done a bad job. The painter demanded payment, but the owner told him that the paint job was poor and refused to pay. The painter pleaded that he was desperately in need of money. The owner told the painter that she would pay him $4,500, provided he repainted the porches. The painter reluctantly agreed, and the owner gave the painter a check in the amount of $4,500. The painter went to his bank, indorsed the check "under protest" and signed his name, then deposited the check in his account. He never returned to repaint the porches. The painter sues the owner for $500, which he believes is still owed to him on his contract to paint the porches. Will he prevail? •A) Yes, because he indorsed the check "under protest." •B) Yes, but only if he repaints the porches. •C) Yes, because he performed the contract by painting the porches the first time. D) No, even if he repaints the porches.

D

•. A widget seller negotiates with a buyer for the purchase of 100 widgets for $10,000, with delivery 30 days from the signing of the agreement. After the parties sign the agreement, which contains a merger clause, the buyer asks the seller if delivery could be in 20 days and the seller says, "Yes, I promise we will do that." The seller does not deliver until 30 days from signing. The buyer sues the seller and the seller seeks to bar the evidence that the seller promised delivery in 20 days. Will the evidence be excluded under the parol evidence rule? •A. Yes, because it contradicts a term of the written agreement. •B. Yes, because it supplements a written agreement that is complete and exclusive. •C. No, because it is offered for the purpose of interpreting the writing. •D. No, because it is offered on an issue of modification.

D.

•6. A consumer sees a television set for sale at a retail store with a price of $800. After she buys the set, she discovers that another store is selling the same set for a price of $400. She complains, but the store refuses to do anything. What is consumer's best argument to avoid this contract? •A. The seller did not act in good faith in setting a price twice as high as the market price. •B. It is unconscionable to sell a television set at a price twice as high as others are selling it for. •C. The price was not within the consumer's reasonable expectations. •D. None of the above is a good argument.

D.

•Seller and Buyer enter into an agreement by which Seller agrees to sell her 1964 Ford Mustang to Buyer for $50,000. Both parties believe the car is worth $50,000. Two months after the sale, Buyer discovers that the car is worth only $10,000. As a result, Buyer brings suit to rescind the contract. What is the likely outcome of such litigation? •A. Buyer will prevail because Buyer mistakenly believed the car was worth $50,000. •B. Buyer will prevail because Seller mistakenly believed car was worth $50,000. •C. Buyer will prevail because both Buyer and Seller mistakenly believed the car was worth $50,000. •D. Seller will prevail.

D.

Promissory Fraud

If a fraud claim is based on a promise of future action, it is actionable if the promise was made with an intent not to perform the future action

Perfect Tender rule

If goods, or tender of delivery, fail to conform to the contract, the Buyer has the right to: Accept the goods Reject the entire shipment Accept part and reject part

consideration adequacy

If the requirement of sufficiency of consideration is met there is no additional requirement of equivalence in the values exchanged

Prevention and Cooperation

It is a general principle of contract law that if one party to a contract wrongfully hinders, prevents, or makes impossible performance by the other party, the latter's failure to perform will be excused A contract imposes upon each party a duty of good faith and fair dealing in its performance and its enforcement

if there is an enforceable deal, what are its terms?

Parol evidence rule, contract interpretation, gap fillers

Two types of unconscionability

Procedural: concerns the contract formation process, including high pressure sales tactics, inequality of bargaining power,a adhesion contracts, deceptive practices and language in the contract, and an imbalance in the understanding and aceumen of the parties Substantive: concerns the content of the contract; onsided, overly harsh, shocking to the conscience

Part performance; SOF

Real estate: in a majority jurisdictions, a contract for the transfer of land is enforceable by specific performance if the buyer makes full payment and either takes possession of the land or makes substantial improvement Theron. goods: a contract which does not satisfy the requirements of subsection 1 but which is valid in other respects is enforceable a. if the goods are to be specifically manufactured for the buyer and are not suitable for sale to others in the ordinary course of the seller's business and the seller, before notice of repudiation is received and under circumstances which reasonably indicate that the goods are for the buyer, has made either a substantial beginning of their manufacture or commitments for their procurement; or c. with respect to goods for which payment has been made and accepted or which have been received and accepted Services: part performance of a service contract will not satisfy the sof (but quasi-contract damages may be available to part performer). Full performance by either party satisfies sof.

UCC (article 2) or Common law

Solely goods- UCC non goods- common law

when does an advertisement or solicitation constitute an offer

Test: 1. does the language of the ad contain a promise? 2. Are the terms of the promise certain and definite? 3. Is the offer clearly identified?

Mixed purpose test

What is the predominant purpose? 1. the language of the parties contract 2. the nature of the business of the supplier of the goods and non goods (service) 3. the reason the parties entered into the contract 4. the respective amounts charged under the contract for goods and for non goods

Frustration of Purpose

Where, after a contract is made, a party's principal purpose is substantially frustrated without his fault by the occurrence of an event the nonoccurrence of which was a basic assumption on which the contract was made, his remaining duties to render performance are discharged, unless the party has assumed the risk * Unexpected circumstances undermine the purpose of the contract *

Parol Evidence Rule

a binding integrated agreement discharges prior agreements to the extent that it is inconsistent with them a binding completely integrated agreement discharges prior agreements to the extent that they are within the scope

Other party's breach UCC

a buyer's acceptance occurs when the buyer a. after reasonable opportunity to inspect the goods signifies to the seller that the goods are conforming or that he will take or retain them in spite of their non conformity b. fails to make an effective rejection, but such acceptance does not occur until the buyer has had a reasonable opportunity to inspect them; or c. does any act inconsistent with the seller's ownership Once goods are accepted, the buyer may no longer reject the goods. After acceptance, the buyer must pay for the goods and if, the goods are nonconforming, the buyer's options are: a. assert a claim for breach of warranty revoke acceptance The buyer may revoke acceptance if the non-conformity substantially impairs its value to him and he accepted the goods a. on the reasonable assumption that the nonconformity would be cured and it has not been seasonably cured b. without discovery of such nonconformity if his acceptance was reasonably induced either by the difficulty or discovery before acceptance or by the seller's assurances

Assumption of the risk

a party bears the risk of mistake when A. the risk is allocated to him by agreement of the parties, or B. he is aware, at the time the contract is made, that he has only limited knowledge with respect to the facts to which the mistake relates but treats his limited knowledge as sufficient, or C. the risk is allocated to him by the court on the ground that it is reasonable in the circumstances to do so

non disclosure

a person's nondisclosure of a fact known to him is equivalent to an assertion that the fact does not exist in the following cases only: -(a) where he knows that disclosure of the fact is necessary to prevent some previous assertion from being a misrepresentation or from being fraudulent or material. -(b) where he knows that disclosure of the fact would correct a mistake of the other party as to a basic assumption on which that party is making the contract and if non-disclosure of the fact amounts to a failure to act in good faith and in accordance with reasonable standards of fair dealing. -(c) where he knows that disclosure of the fact would correct a mistake of the other party as to the contents or effect of a writing, evidencing or embodying an agreement in whole or in part. -(d) where the other person is entitled to know the fact because of a relation of trust and confidence between them.

Marriage

a promise for which all or part of the consideration is either marriage or a promise to marry is within the sof, except in the case of an agreement which consists only of mutual promises of two persons to marry each other.

rejection by the offeree

an offeree's power of acceptance is terminated by her rejection of the offer, unless the offeror has manifested a contrary intention an option remains open for the prescribed time even though the holder of the option indicated during such time period that he does not intend to exercise it. However, if the other party relies on the rejection, the option ends.

what is an option contract

an option contract is a promise which meets the requirements for the formation of a contract and limits the promisor's power to revoke an offer.

Mistake

a. misunderstanding (a real defense) b. mutual mistake (an affirmative defense) c. unilateral mistake d. reformation (remedy for mistake)

Concealment

action intended or known to be likely to prevent another from learning a fact is equivalent to an assertion that the fact does not exist

Fraud

affirmative fraud, promissory fraud, concealment, non-disclosure

Novation

all parties to the original contract (and a third party) agree to substitute the new third party for one of the original parties. In such case, the original party's (the party who was replaced) performance is excused and the. new party assumes that duty

Accord and Satisfaction

an agreement by the parties to contract to accept a different type of performance by one party and that party so performs. the accord suspends the original performance; the satisfaction excuses the original duty. If there is no satisfaction, the other party may sue on the original agreement or the accord If a creditor agrees to take less than full payment in exchange for a release, may the creditor seek the remainder from the debtor?

Mutual rescission

an agreement by which each party agrees to discharge all of the other party's remaining duties of performance under an existing contract. A mutual recession excuses performance only if both parties' performances are still executory.

Illusory promises

an illusory promise is one which by its terms makes performance entirely optional for the promisor

is the written contract integrated?

an integrated agreement is a writing or writings constituting a final expression of one or more terms of an agreement whether there is an integrated agreement is to be determined by the court as a question preliminary to determination of a question of interpretation or application of the parol evidence rule where the parties reduce an agreement to a writing which in view of its completeness and specificity reasonably appears to be a complete agreement, it is taken to be an integrated agreement unless it is established by other evidence that the writing did not constitute a final expression

Merchant's firm order

an offer by a merchant to buy or sell goods in a signed writing which by its terms gives assurance that it will be held open is not revocable, for lack of consideration, during the time stated or if no time is stated for a reasonable time, but in no event may such period of irrevocability exceed three months

Lapse of time

an offeree's power of acceptance is terminated at the time specified in the offer or, if no time is specified, at the end of a reasonable period.

•A consumer buys a candy bar (Almond Joy) from a grocery store, bites into it, and breaks a tooth on a piece of metal imbedded in the candy bar. Does the consumer have a claim for breach of warranty against the store? •A. Yes, because a candy bar that breaks a tooth is probably not merchantable. •B. No, because the consumer assumed the risk of such injury. •C. No, because if anyone is to blame, it is the manufacturer and not the seller. •D. No, because no promises were made about the candy bar. • •Would your answer change if the consumer broke the tooth on a stale almond?

answer A

On May 1, a sales manager announced at a monthly sales meeting that he would give a $1k bonus to the salesperson with the highest sales at the end of the month. The sales manager's announcement would be best interpreted as: A. Prelim negotiations B. A promise to make a conditional gift C. A contractual offer, creating a power of acceptance D. A mere statement designed to induce the salesperson to work harder and boost sales

answer: C

•1. During negotiations to purchase a used car, a buyer asked a dealer whether the car had ever been in an accident. The dealer replied: "It is a fine car and has been thoroughly inspected and comes with a certificate of assured quality. Feel free to have the car inspected by your own mechanic." In actuality, the car had been in a major accident, and the dealer had repaired and repainted the car, successfully concealing evidence of the accident. The buyer declined to have the car inspected by his own mechanic, explaining that he would rely on the dealer's certificate of assured quality. At no time did the dealer disclose that the car had previously been in an accident. The parties then signed a contract of sale. After the car was delivered and paid for, the buyer learned that the car had been in a major accident. If the buyer sues the dealer to rescind the transaction, is the buyer likely to succeed? •(A) No, because the buyer had the opportunity to have the car inspected by his own mechanic and declined to do so. •(B) No, because the dealer did not affirmatively assert that the car had not been in an accident. •(C) Yes, because the contract was unconscionable. •(D) Yes, because the dealer's statement was intentionally misleading and the dealer concealed evidence of the accident.

answer: D

Oral contracts

are generally enforceable, but six types of contracts are subject to the SOF A. marriage B. year-term of more than one year C. land transaction D.Executor/ Administrator contracts E. goods--sale of goods for $500 or more F.Suretyship contracts my legs

Punitive damages

are not recoverable for a breach of contract unless the conduct constituting the breach is also a tort for which punitive damages are recoverable

reliance damages

as an alternative to expectation damages, the injured party has a right to damages based on his reliance interest, including expenditures made in preparation for performance or in performance, less any loss that the party in breach can prove with reasonable certainty the injured party would have suffered had the contact been performed

•Jeffrey's employer is sending him on a six-month overseas business trip. Not wanting to disrupt his 12-year-old son Kyle's school year, Jeffrey forms a contract with Kandy under which Kandy promises to live in Jeffrey's house as a full-time caregiver to Kyle, in exchange for which Jeffrey promises to pay Kandy $35,000 for the full six-month period. If Kandy fails properly to care for Kyle, can Kyle (through his guardian ad litem) sustain an action against her? •A. Yes, because Jeffrey and Kyle have a direct familial relationship •B. Yes, because contract law affords Kyle that right. •C. No, because Kyle is not a party to the contract between Jeffrey and Kandy •D. No, because Jeffrey was not within the jurisdiction at the time of the breach.

b

Seller has goods

breach occurs when: seller fails to deliver; buyer rightfully rejects goods and returns them; buyer rightfully revokes acceptance of goods and returns them; good destroyed before risk of loss shifts to buyer Damages: a. cover--difference between cover price and contract price b. difference between market value and contract price c. incidental damages and d. consequential damages minus e. expenses saved by breach equitable relief: specific performance (unique goods only)

buyer breaches, seller has goods

breach occurs when: buyer repudiates before delivery; buyer wrongfully rejects; buyer wrongfully revokes acceptance damages: difference between resale price or market value and contract price; plus incidental damages minus expenses saved damages lost volume seller: profit; goods the seller is unable to resell or the circumstances reasonably indicate that an effort to resell the goods will be unavailing: contract price

•The owner of a summer house entered into a written agreement with a plumber. The contract contained a clause requiring all plumbing work to be completed by noon on June 1 and provided that the homeowner would pay the plumber $1,200 for his work. The plumber began working on the job on May 28. When he quit working for the day on the afternoon of May 29, half of the job was completed. Later that evening, the house was swept away in a flash flood. Which of the following best describes the obligations of the plumber and the homeowner after the flood? •A. Neither the plumber nor the homeowner is discharged from their obligations under the contract. •B. The homeowner is obliged to pay the plumber $1,200. •C. The plumber is discharged from his obligation but is entitled to recover from the homeowner the fair value of the work he performed prior to the flood. •D. Neither the plumber nor the homeowner has any further obligations.

c

Expectation damages--special class

construction contracts: if a breach results in defective or unfinished construction, the plaintiff may recover damages based on a. the diminution in the market price of the property caused by the breach, or b. the reasonable cost of completing performance or of remedying the defects if that cost is not clearly disproportionate to the probable loss in value to him Employment contracts: if an employee breaches an employment contract, the employer is entitled to recover the difference between the wages paid to the replacement employee and the breaching employee's contract. If an employer breaches an employment contract, the employee is entitled to recover the full contract price (plus incidental damages), minus: -the salary the employee would have received if the employee had made reasonable efforts to obtain a position of the same rank; in the same type of work; in the same location -the actual salary the employee received from any job Contracts for the sale of land: the standard measure of damages for breach of a land sale contract is the difference between the contract price and the fair market value of the land.

Land transactions

contracts to transfer, buy, or pay fo an interest in land are within the SOF, except for short term leases.

•A homeowner and a local builder entered into a written contract that called for the builder to build a second story onto the top of the homeowner's one-story residence. When scheduling conflicts arose, the builder asked the homeowner if they could substitute his buddy, an out-of-town builder who had comparable experience and skills, to perform the local builder's part of the contract. All of the parties agreed to the substitution. Unfortunately, the out-of-town builder made a major blunder that will be quite expensive to correct. Is the local builder liable to the homeowner for the cost of correcting the defect? •A. Yes, because the substitution in and of itself does not relieve the local builder of liability on the underlying contract. •B. Yes, because the local builder did not give any consideration on which to base a release. •C. No, because the local builder delegated his duties to the out-of-town builder. •D. No, because the local builder was discharged through a novation.

d

Anticipatory Repudiation

if one party unambiguously declares that it will not perform prior to the date of performance, the other party's performance is excused and that party has to choices: 1. await performance by the repudiating party for a commercially reasonable time or 2. treat the declaration as an immediate breach even if it has notified the repudiating party that it would await performance and has urged retraction. If the repudiating party retracts the repudiation before the nonrepudiation party, party has materially changed position in reliance on the repudiation or has notified the repudiating party that the contract is terminated, the obligations of both parties are revived.

misunderstanding

if the parties attach materially different meanings to their manifestations and neither party knows or has reason to know the meaning attached by the other; or each party knows or each party has reason to know the meaning attached by the other

Illegality

if the subject matter of the contract is illegal, the contract is void and not enforceable

What does not constitute consideration

illusory promises past consideration (exceptions: statute of limitations, infancy cases, restatement of contracts section 86)

If there is an enforceable deal, what are the terms?

implied terms: at common law, if the parties failed to agree on a material term, a court would not enforce the agreement becuase of indefiniteness Under modern law, if a contract has been formed that fails to address one or more terms, the court will fill-in those terms: 1. good faith 2. fair dealing 3. reasonable efforts UCC has gap fillers for 1. time and place of payment 2. place of delivery 3. price 4. date of delivery 5. mode of delivery

Subsequent even

impossibility Commercial impracticability frustration of purpose

Incapacity

infancy incompetency intoxication

Acceptance

is defined as a manifestation of assent to the terms of the offer made by the offeree in the manner invited or required, if any, by the offer.

Specific performance

may b e decreed where the goods are unique or in other proper circumstances, such as where the buyer is unable to cover or is unable to secure a comparable long-term contract specific performance will not be decreed against a vendor, where he has conveyed the property of goods to a bona fide purchaser

merchant

means a person who deals in goods of the kind or otherwise by his occupation holds himself out as having knowledge or skill peculiar to the practices or goods involved in the transaction or to whom such knowledge or skill may be attributed by his employment of an agent or broker or other intermediary who by his occupation holds himself out as having such knowledge or skill

subsequent agreement

novation, accord and satisfaction, mutual rescission

deal has three components

offer (valid) acceptance timel acceptance

damages

once a court has ruled that a breach occurred the non breaching party may choose to pursue various damage theories, including expectation, reliance, and restitution

Duress and Undue influence

physical, economic, undue influence

Gap Filler--place of delivery and risk of loss

place of delivery: if the contract does not call for (expressly or implicitly) delivery, the UCC presumes that the buyer will pick up such goods at the seller's place of business or, if non, the seller's residence.

Reliance damages (continued)

reliance damages are backward-looking damages; their purpose is to put the non-breaching party in the position it would have been in had the contract never been formed. -reliance damages are generally awarded in cases where the non-breaching party cannot establish expectation damages with reasonable certainty; in those cases, the non-breaching party has a right to damages based on its reliance interest, including reasonable expenditures made in preparation for performance. -reliance damages are also the traditional remedy in promissory estoppel cases

Affirmative defenses

rest with the defendant to prove Incapacity Statute of frauds Illegality Duress and undue influence Fraud Mistake Unconscionability

Sale of goods for $500 or more

some writing to indicate that a contract of sale has been made between the parties

Equitable Remedies

specific performance and injunction

Constructive conditions

what if the contract is silent on the order of performance? 1. where all or part of the performances can be rendered simultaneously, they are due simultaneously, unless the language or the circumstances indicate otherwise Where the performance of only one party under an exchange requires a period of time, that party's performance is due at an earlier time than that of the other party, unless the language or the circumstances indicate otehrwise Strict compliance is required for express conditions. By contrast, constructive conditions may be satisfied by substantial performance (or substantial compliance), thus enabling the party who substantially performed to bring suit on the contract for the full price less any damages caused by the failure to fully perform

Restitution

when there is no contract; for breach of contract; for breaching plaintiff; when a contract is unenforceable

Mutual Mistake

where a mistake of both parties at the time a contract was made as to a basic assumption on which the contract was made has a material effect on the agreed exchange of performances, the contract is voidable by the adversely affected party unless he or she bears the risk of the mistake.

Unilateral Mistake

where a mistake of one party at the time a contract was made as to a basic assumption on which he made the contract has a material effect on the agreed exchange of performance that is adverse to him, the contract is voidable by him if he does not bear the risk of the mistake and the effect of the mistake is such that enforcement of the contract would be unconscionable or the other party had reason to know of the mistake or his fault caused the mistake

Reformation

where a writing that evidences or embodies an agreement in whole or in part fails to express the agreement becuase of a mistake of both parties as to the contents of the writing or fraud, the court may at the request of a party reform the writing to express the agreement, except to the extent that rights of third parties such as good faith purchasers for value will be unfairly affected.

Illegality

where after the making of an offer and before acceptance the proposed contract becomes illegal the offer is terminated an option contract is also terminated by an intervening illegality

Term of more than one year

where any promise in a contract cannot be fully performed within a year from the time the contract is made, all promises in the contract are within the sof.

A orally promises to work for B, and B promises to employ A for five years at a stated salary. Are the promises within the SOF? My legs

yes, within the SOF If the facts were different, and the term was not certain, it would not be within the SOF.

a burglar stole Collecta's impressionist painting valued at $400,000. collects, who had insured the painting for $300k with artistic insurance co, promised to pay $25k to Snoop, a full-time investigator for artistic, if he effected the return of the painting to her in good condition. By company rules, Artistic permits its investigators to accept and retain awards from policyholders for the recovery of insured property. Snoop, by long and skillful detective work, recovered the picture and returned it undamaged to Collect. If collects refuse to pay Snoop, and he sues her for $25k, what is the probable result under the prevailing modern rule?

•(A) Collecta wins, because Snoop owed Artistic a preexisting duty to recover the picture if possible. •(B) Collecta wins, because Artistic, Snoop's employer, has a preexisting duty to return the recovered painting to Collecta. •(C) Snoop wins, because Collecta will benefit more from return of the $400,000 painting than from receiving the $300,000 policy proceeds. •(D) Snoop wins, because the preexisting duty rule does not apply if the promisee's (Snoop's) duty was owed to a third person. Answer: D

A client consulted a lawyer about handling the sale of the client's building, and asked the lawyer what her legal fee would be. The lawyer replied that her usual charge was $100 per hour, and estimated that the legal work on behalf of the client would cost about $5,000 at that rate. The client said, "Okay; let's proceed with it," and the lawyer timely and successfully completed the work. Because of unexpected title problems, the lawyer reasonably spent 75 hours on the matter and shortly thereafter mailed the client a bill for $7,500, with a letter itemizing the work performed and time spent. The client responded by a letter expressing his good-faith belief that the lawyer had agreed to a total fee of no more than $5,000. The client enclosed a check in the amount of $5,000 payable to the lawyer and conspicuously marked, "Payment in full for legal services in connection with the sale of [the client's] building." Despite reading the "Payment in full ..." language, the lawyer, without any notation of protest or reservation of rights, endorsed and deposited the check to her bank account. The check was duly paid by the client's bank. A few days later, the lawyer unsuccessfully demanded payment from the client of the $2,500 difference between the amount of her bill and the check, and now sues the client for that difference. What, if anything, can the lawyer recover from the client? A. Nothing, because the risk of unexpected title problems in a real-property transaction is properly allocable to the seller's attorney and thus to the lawyer in this case. B. Nothing, because the amount of the lawyer's fee was disputed in good faith by the client, and the lawyer impliedly agreed to an accord and satisfaction. C. $2,500, because the client agreed to an hourly rate for as many hours as the work reasonably required, and the sum of $5,000 was merely an estimate. D. The reasonable value of the lawyer's services in excess of $5,000, if any, because there was no specific agreement on the total amount of the lawyer's fee.

•A) Nothing, because the risk of unexpected title problems in a real-property transaction is properly allocable to the seller's attorney and thus to Lawyer in this case. •(B) Nothing, because the amount of Lawyer's fee was disputed in good faith by Client, and Lawyer impliedly agreed to an accord and satisfaction. •(C) $2,500, because Client agreed to an hourly rate for as many hours as the work reasonably required, and the sum of $5,000 was merely an estimate. •(D) The reasonable value of Lawyer's services in excess of $5,000, if any, because there was no specific agreement on the total amount of Lawyer's fee. Answer: B

John and Margee work as coal minor. one day while working, Margee noticed that a dynamite charge was not properly placed and that John was at risk of death. Margee rushed over to rush John out of the way. In doing so, Margie absorbed sue of the blast and was crippled. John was so moved by the sacrifice that he promised to pay Margie $1k per month for the rest of her life. John paid Margie for 5 years, but then he died. John's estate does not want to continue payments. which is most accurate?

•A. Margee will definitely not be allowed to enforce Jon's promise because it is supported only by past consideration. •B. Margee might be allowed to enforce Jon's promise, but only if it was reduced to writing. •C. Margee will definitely not be allowed to enforce Jon's promise because it was an offer to make a gift, which offer was automatically revoked upon Jon's death. •D. Margee might be able to enforce the agreement if the court accepts the validity of the material benefit rule. Answer D

Tenant rented a commercial building from Landlord, and operated a business in it. The building's large front window was smashed by vandals six months before expiration of the Tenant-Landlord lease. Tenant, who was obligated thereunder to effect and pay for repairs in such cases, promptly contracted with Glazier to replace the window for $2,000, due 30 days after satisfactory completion of the work. Sixty days later, Tenant mailed a $1,000 check to Glazier bearing on its face the following conspicuous notation: "This check is in full and final satisfaction of your $2,000 window replacement bill." Without noticing this notation, Glazier cashed the check and now sues Tenant for the $1,000 difference. If Tenant's only defense is accord and satisfaction, is Tenant likely to prevail? A. No, because Glazier failed to notice Tenant's notation on the check. B. No, because the amount owed by Tenant to Glazier was due and undisputed. C. Yes, because by cashing the check Glazier impliedly agreed to accept the $1,000 as full payment of its claim. D. Yes, because Glazier failed to write a reservation-of-rights notation on the check before cashing it.

•A. No, because Glazier failed to notice Tenant's notation on the check. •B. No, because the amount owed by Tenant to Glazier was liquidated and undisputed. •C. Yes, because by cashing the check Glazier impliedly agreed to accept the $1,000 as full payment of its claim. •D. Yes, because Glazier failed to write a reservation-of-rights notation on the check before cashing it. Answer: B

On June 1, Buyer, inc., a widget Manu, entered into a written agreement with Mako, inc., a tool maker, in which mako agreed to produce and sell buyer 12 sets of newly designed dies to be delivered august 1 for the price of $50,000, payable ten days after deliver. Encountering unexpected expenses in the purchase of special alloy steel required for th edges, Mako advised Buyer that production costs would exceed the contract price; and on July 1 buyer and Mako signed a modification to the June 1 agreement increasing the contract price to $60,000. After timely receipt of 12 sets of dies conforming to the contract specifications, buyer paid Mako $50,000 but refused to Pau more. Is mako entitled to 10,000, for breach of Buyem's July promise?

•A. No, because Mako provided no new consideration for the July 1 agreement. •B. No, because Mako had a pre-existing duty to perform. •C. Yes, if Mako was acting in good faith. •D. Yes, but only if Mako agreed to deliver the dies prior to August 1. Answer: C

•3. A gambler owed his uncle $9,000, which was due on January 1. On January 15, the gambler offered to pay the uncle $8,000 if he would agree to accept the amount in full satisfaction of the $9,000 debt. The uncle agreed and the gambler paid him the $8,000. If the uncle then sues the gambler for $1,000, the uncle will:

•A. Win, because the gambler had an obligation to pay $9,000 on January 1. •B. Lose, because of the uncle's agreement to accept $8,000. •C. Lose, because there was an accord and satisfaction. •D. Lose, because the uncle agreed to the $8,000 after the January 1 due date. Answer: A

•2. In a state where gaming is legal, a professional gambler ran up a tab of $50,000 at his favorite casino. Pursuant to a longstanding agreement between the gambler and the casino, once the gambler's tab reached $50,000 he was required to repay the debt in five monthly installments of $10,000 before putting any additional charges on his tab. After making three repayments, the gambler approached the casino owner and offered an immediate payoff of $15,000 in cash as payment in full. The casino owner had a cash flow problem and needed the money, so he agreed. The gambler made the cash payment of $15,000 that same day. A few days later, the casino owner demanded $5,000 from the gambler. Does the casino owner have a right to collect $5,000 from the gambler?

•A. Yes, because the gambler had a preexisting duty to pay the full $50,000. •B. Yes, because the casino owner acted under duress when he accepted the immediate payoff of $15,000 in cash as payment in full for the gambler's debt. •C. No, because there was a discharge by release. •D. No, because there was an accord and satisfaction. Answer: D

Failure of a condition precedent--express conditions of satisfaction

•If a contracting party's performance is expressly conditioned on his or her "satisfaction" with the other party's performance, the following rules apply: -if the contract is for services that do not require personal taste or aesthetics (e.g., a contract to install a furnace), the party (whose performance is conditioned on his or her satisfaction) must act reasonably (objective test) in determining whether the other party's performance is satisfactory; in close cases, courts will apply this objective test -if the contract is for services that require personal taste or aesthetics (e.g. a contract with an artist or a contract with an interior decorator), the party (whose performance is conditioned on his or her satisfaction) must act in good faith (subjective test) in determining whether the other party's performance is satisfactory -if a third party (e.g., an architect) is to make the determination of "satisfactory" performance, the third party must act in good faith (subjective test)

Other party's breach --common law

•If one party is in material breach (i.e., has not substantially performed), the other party's duty to perform (i.e., pay) is discharged. -In such cases, the breaching party may not sue on the contract; the breaching party is limited to a remedy in quasi-contract, with damages measured by the excess of benefit conferred over and above the harm caused. •If the breach is not material, the breaching party may sue on the contract with any defects in performance recompensed by a setoff for the damages caused.

Impossibility (common law)

•Where, after a contract is made, a party's performance is made impracticable without his or her fault by the occurrence of an event the non-occurrence of which was a basic assumption on which the contract was made, his or her duty to render that performance is discharged, unless the language or the circumstances indicate the contrary. •B. If the existence of a particular person is necessary for the performance of a duty, his or her death or such incapacity as makes performance impracticable is an event the non-occurrence of which was a basic assumption on which the contract was made. •C. A party's failure to render or to offer performance because of an impossibility or severe impracticability excuses that party's performance, unless that party assumed the risk that he or she would have to perform despite such a failure. -If, at the time of contracting, the parties were aware that an event was not certain to occur, that event may not be used to support a claim of impossibility or impracticability.


Kaugnay na mga set ng pag-aaral

Nurses Role for Blood Transfusion

View Set

HA: Assessing children and adolescents

View Set